Trauma Flashcards

1
Q
  1. A 27-year-old man presents with posterior instability of the shoulder. Transaxial ts T1 MR arthrogram shows posterioinferior labrum tear and tear of the posterior scapular periosteum. Which is the diagnosis?

A. Reverse Bankart lesion

B. Bankart lesion

C. Bennett lesion

D. Posterior labrocapsular periosteal sleeve avulsion

E. Humeral avulsion of the posterior glenohumeral Ligament

A

A. Reverse Bankart lesion

The posterior inferior labrum is detected from glenohumeral attachment and there is an avulsion tear of the posterior scapular periosteum.

How well did you know this?
1
Not at all
2
3
4
5
Perfectly
2
Q
  1. A 20-year-old long-distance runner has a several week history of right lower leg pain. A plain film was reported as normal but CT showed multiple areas of osteopenia and cavities of the anterior tibial cortex. Which is the diagnosis?

A. Stress fracture

B. Chronic external compartment syndrome

C. Medial tibial stress syndrome

D. Periostitis

E. Interosseous membrane injury

A

C. Medial tibial stress syndrome

A spectrum of lesions can occur from repetitive stress. These include periostitis, cortical osteopenia, cancellous bone and cortical fractures. Injuries are most frequently in the cortex of the distal 2⁄3 of the tibia and known as medial tibial stress syndrome.

How well did you know this?
1
Not at all
2
3
4
5
Perfectly
3
Q
  1. A 27-year-old athlete develops groin pain. A pelvic radiograph suggests osteitis pubis. Which is the single best answer?

A. Osteitis pubis usually demonstrates rapid radiographic changes

B. Is due to a single traumatic insult in most cases

C. Is associated with inflammatory mediated inappropriate osteoblsatic activity

D. Is associated with pubic or perineal pain with resisted hip abduction

E. Radiographic changes include alternating osteopenia and sclerosis

A

E. Radiographic changes include alternating osteopenia and sclerosis

This is a self-limiting but often protracted condition secondary to repetitive microtrauma with osteoclastic activity and osseous resorption.

Radiographs show irregularity of subchondral bone plate, erosions, fragmentation and alternating osteopenia and sclerosis.

It may lead to joint space widening.

How well did you know this?
1
Not at all
2
3
4
5
Perfectly
4
Q

10) A 28-year-old physically active young man undergoes a hip MR arthrogram for chronic pain that is worse during exercise. There is a history of several months of hip pain when the patient was a teenager that was not investigated. Images show a loss of the femoroacetabular sulcus superiorly with an associated acetabular labral tear. What is the underlying condition?

a. pincer femoroacetabular impingement

b. cam femoroacetabular impingement

c. combined femoroacetabular impingement

d. traumatic labral tear

e. osteochondritis dissecans

A

b. cam femoroacetabular impingement

Cam is the most common form of femoroacetabular impingement in men, typically presenting in the third or fourth decade.

It is often related to a previous slipped upper femoral epiphysis in the teenage years.

A change in the rotational axis (increase in the alpha angle) causes the proximal superior femoral neck to impinge upon the superior acetabular margin and labrum, in turn causing intermittent pain, particularly in physically active individuals.

Even without a history of slipped femoral epiphysis, an osseous bump on the superior femoral neck obliterating the femoroacetabular sulcus can cause symptoms.

Labral or articular cartilaginous tears can follow repetitive microtrauma, leading to persistent pain and locking.

pincer type is common in women and is caused by an abnormally deep acetabulum.

How well did you know this?
1
Not at all
2
3
4
5
Perfectly
5
Q

@# 14) In the spectrum of perilunate ligamentous injuries and instability, volar tilt of the lunate, seen as a triangular or ‘pie-shaped’ lunate on the AP projection of the wrist, is most commonly a feature of which of the following?

a. scapholunate dissociation

b. perilunate dislocation

c. lunate dislocation

d. volar intercalated segmental instability

e. dorsal intercalated segmental instability

A

c. lunate dislocation

The lesser arc refers to the arc of ligamentous attachments around the lunate.

These ligaments become disrupted in a stepwise four-stage fashion.

Stage I injury is to the scapholunate ligament, leading to dissociation with rotary subluxation of the scaphoid.

Stage II is radiographically characterized by perilunate dislocation, caused by additional injury to the capitolunate joint. The carpus migrates dorsally and the lunate maintains a normal relationship with the radius.

Stage III involves the triquetrolunate ligaments,

and stage IV is complete disruption of the perilunate ligaments, allowing dislocation and rotation of the lunate. It is this rotation that creates the triangular outline on AP radiographs. Segmental instabilities relate to the spectrum of dynamic scaphoid instability.

How well did you know this?
1
Not at all
2
3
4
5
Perfectly
6
Q

30) MRI of the knee in an 18-year-old man, performed for pain and limited joint movement, reveals an osteochondral lesion of the medial femoral condyle. Other than displacement, which MRI finding is the most specific indication of an unstable osteochondral fragment?

a. joint effusion

b. subfragmental bone resorption

c. 3 mm cyst deep to the lesion

d. underlying linear high signal on T2W images

e. multiple lesions

A

d. underlying linear high signal on T2W images

High T2 signal in the bone underlying an osteochondral lesion has been described as the most common of four MRI findings indicating instability of an osteochondral fragment, which is the most important factor when considering treatment options.

The reported accuracy of this sign for predicting instability varies from 45% to 85%, with one study reporting an increased accuracy when this sign is combined with the second sign of a cartilaginous defect on T1W images.

However, another study states that often only a single indicator is present.

The other indicators of instability are high signal in the articular cartilage and a cystic lesion in the bed (but this needs to be 5 mm or larger).

How well did you know this?
1
Not at all
2
3
4
5
Perfectly
7
Q

@# 33) A 23-year-old man falls onto his outstretched right hand with his elbow flexed. AP and lateral radiographs of the mid-forearm reveal a fracture of the middle third of the radius. Which additional radiograph should be performed?

a. clavicle

b. shoulder

c. elbow

d. oblique forearm

e. wrist

A

e. wrist

A Galeazzi fracture–dislocation is a pattern of injury sustained by falling on an outstretched hand with a flexed elbow. It most commonly consists of a fracture of the radial diaphysis with dislocation or subluxation of the distal radioulnar joint. It is associated with a high rate of non-union, and one or both components are usually treated with surgical fixation.

It is important therefore that the radiologist can recognize potential patterns of injury and radiographically demonstrate their full extent. As a general rule, fractures should be viewed in two orthogonal planes, as should the joint above and below any fracture.

The opposite pattern, of an ulnar shaft fracture with dislocation of the proximal radial head, is termed a Monteggia fracture–dislocation.

A mnemonic for remembering the two is Glasgow Rangers (Galeazzi, radius) and Manchester United (Monteggia, ulna), which indicates for each injury which of the forearm bones is fractured.

How well did you know this?
1
Not at all
2
3
4
5
Perfectly
8
Q

@# 48) A young male patient sustains an external rotational injury to his left ankle and is unable to bear weight. A plain radiograph of the ankle performed in accident and emergency shows no fracture but does show soft-tissue swelling over the medial malleolus and widening of the ankle joint space medially (lateral talar shift). Which of the following additional view(s) should be performed?

a. mortise view

b. calcaneus

c. foot

d. knee

e. contralateral ankle

A

d. knee

The Maisonneuve fracture is a spiral fracture of the upper third of the fibula associated with a tear of the distal tibiofibular syndesmosis and the interosseous membrane.

The medial component of the injury may be an associated fracture of the medial malleolus or rupture of the deep deltoid ligament.

The ankle joint is effectively a bony ring that extends up to the knee.

Interruption of the ring in this way allows lateral displacement of the fibula and so disruption of the congruence of the ankle mortise, resulting in an unstable ankle injury that requires surgical fixation.

How well did you know this?
1
Not at all
2
3
4
5
Perfectly
9
Q

@# 51) At which of the following skeletal locations does avascular osteonecrosis typically only occur in the presence of an associated fracture?

a. medial tibial condyle

b. second metatarsal head

c. lunate

d. femoral head

e. proximal scaphoid pole

A

e. proximal scaphoid pole

Osteonecrosis may be caused by two mechanisms: interruption of arterial supply, and intra- or extra-osseous venous insufficiency.

Interruption of vascular supply is usually associated with a fracture, as seen in the proximal scaphoid following waist fractures.

Femoral head osteonecrosis can occur with subcapital fractures, or without fracture as in Legg–Calve´ –Perthes disease.

Other common locations that may develop osteonecrosis without overt fracture include the medial tibial condyle (Blount’s disease), metatarsal head (Freiberg’s infraction) and the lunate (Kienbock’s disease).

Radiographic findings often lag several months behind the injury or onset of symptoms, and MR is the most sensitive imaging modality.

Radiographic signs include focal radiolucencies, sclerosis, bone collapse and loss of joint space.

How well did you know this?
1
Not at all
2
3
4
5
Perfectly
10
Q

52) A young adult male sustains an acetabular fracture in a high-speed road traffic collision. Which type of acetabular fracture is most commonly associated with significant neurological injury?

a. posterior rim/wall

b. anterior rim/wall

c. transverse T-shape

d. anterior and posterior column

e. central dislocation

A

a. posterior rim/wall

Acetabular fractures are common in multiple or major trauma patients, particularly those involved in road traffic collisions, and are classified according to the Letournel classification.

Fractures are often complex and require accurate delineation with CT, often following limited or suboptimal initial radiographic investigation with or without oblique pelvic (Judet) views.

Isolated posterior rim or wall fractures are the most common type (27%) and are associated with a high frequency of posterior dislocation of the femoral head causing sciatic nerve injury.

If the entire posterior column is involved in the fracture, there is a lower incidence of sciatic nerve injury, as the femoral head may not be dislocated.

Anterior injuries are uncommon (5%) and may be associated with anterior femoral head dislocation and iliac wing fracture.

Transverse fractures account for 9%.

How well did you know this?
1
Not at all
2
3
4
5
Perfectly
11
Q

55) A young adult male sustains an anterior shoulder dislocation while playing rugby. There is no associated fracture. Following apparently uncomplicated reduction in accident and emergency, he is unable to abduct the arm and complains of numbness over the upper lateral arm. What is the most likely cause?

a. supraspinatus tendon tear

b. axillary nerve palsy

c. musculocutaneous nerve palsy

d. shoulder impingement

e. deltoid muscle tear

A

b. axillary nerve palsy

The axillary nerve is a large terminal branch of the posterior cord of the brachial plexus that passes into the posterior aspect of the upper arm via the quadrilateral space, where it winds around the surgical neck of the humerus to supply the deltoid and teres minor muscles.

It has a cutaneous distribution called the ‘regimental badge area’ over the lateral aspect of the deltoid (where a soldier may wear his regimental badge).

Due to its intimate relationship with the humerus and its passage through the relatively small quadrilateral space, the axillary nerve is by far the most commonly injured nerve with shoulder dislocation or fractures.

As loss of abduction may be caused by pain rather than deltoid paralysis,

it is good practice to assess the sensation in the cutaneous distribution of the axillary nerve before and after any attempted shoulder reduction.

How well did you know this?
1
Not at all
2
3
4
5
Perfectly
12
Q

@# 57) Of the following eponyms associated with fractures, which relates to a fracture–dislocation?

a. Segond

b. Jones

c. Smith

d. Barton

e. Hutchinson

A

d. Barton

The use of eponymous names for fractures allows quick and accurate identification and communication of bone injuries while simultaneously alerting clinicians to the potential complications associated with a given fracture pattern. This is also particularly useful when describing complex radiographic appearances to someone remote from the images. The full value of such eponyms depends on accurate use and understanding of their meaning:

Barton’s fracture–dislocation is an intra-articular fracture of the dorsal margin of the distal radius with dorsal dislocation of the radiocarpal joint;

Segond’s fracture is an avulsion fracture of the proximal lateral tibia;

Jones’ fracture is a transverse fracture of the base of the fifth metatarsal, at the junction of the diaphysis and metaphysis;

Smith’s fracture is a distal radial fracture with ventral displacement;

and Hutchinson’s fracture is a triangular fracture of the radial styloid.

How well did you know this?
1
Not at all
2
3
4
5
Perfectly
13
Q

60) Which of the following locations is most often associated with post-traumatic osteolysis?

a. coronoid process of ulna

b. surgical neck of humerus

c. lateral clavicle

d. femoral neck

e. fibular head

A

c. lateral clavicle

The lateral third of the clavicle is the most common location for posttraumatic osteolysis. It is usually preceded by a fairly severe injury to the shoulder, typically dislocation or subluxation of the acromioclavicular joint. Changes may be evident radiographically after as little as 1 month. If no bone loss was apparent on radiographs at the time of injury, the diagnosis is unequivocal. However, if no such comparison can be made, then other causes of lateral clavicular osteolysis include rheumatoid arthritis, scleroderma and hyperparathyroidism.

Other sites affected are the pubic and ischial rami, distal portions of the radius or ulna, the carpus and femoral neck.

Widespread idiopathic osteolysis is termed Gorham’s or vanishing bone disease.

How well did you know this?
1
Not at all
2
3
4
5
Perfectly
14
Q

61) A young man is admitted in cardiac arrest following electrocution. Following successful resuscitation in accident and emergency, he complains of an acutely painful right shoulder with severely decreased range of movement. What is the most likely plain film finding?

a. anterior shoulder dislocation

b. posterior shoulder dislocation

c. acromioclavicular dislocation

d. fractured surgical neck of humerus

e. subacromial impingement

A

b. posterior shoulder dislocation

Posterior shoulder dislocation is much rarer than anterior dislocation, accounting for only 5% of dislocations. It can be caused by direct or indirect force and is most commonly seen following seizure or electrocution.

The internal rotators of the shoulder are stronger than the external rotators, resulting in internal rotation of the arm if all the shoulder muscles contract simultaneously. This internal rotation predisposes to posterior dislocation in the same way that external rotation does for anterior dislocation.

Radiographic findings may be subtle on the AP projection, and include superior position of the humeral head relative to the glenoid, external rotation (the humeral head appears symmetrical like a light bulb), a sharp angle to the scapulohumeral arc and a compression fracture of the anterior humeral head (a reverse Hill–Sachs lesion).

How well did you know this?
1
Not at all
2
3
4
5
Perfectly
15
Q

@# 62) A middle-aged woman falls on an outstretched hand, which becomes immediately painful and swollen. A lateral radiograph shows a small fracture fragment dorsal to the carpus, and the AP radiograph appears normal. Which carpal bone is most likely to be fractured?

a. scaphoid

b. lunate

c. triquetrum

d. capitate

e. hamate

A

c. triquetrum

Carpal fractures in general are much less common than fractures to the distal radius. The two bones most commonly injured are the scaphoid (75%) followed by the triquetrum (14%), and these provide a greater diagnostic challenge radiographically than distal radial fractures.

Triquetrum fractures generally occur on the dorsal surface due to avulsion of the dorsal radiocarpal ligament, or shearing forces from impaction with the ulnar styloid or hamate in hyperextension.

Less commonly, the body of the bone can fracture in a transverse pattern.

A posterior chip fragment can often be seen with dorsal surface fractures, but is only visualized on the lateral view.

Such an injury may be a primary triquetrum injury (such as avulsion) or related to a perilunate fracture–dislocation.

How well did you know this?
1
Not at all
2
3
4
5
Perfectly
16
Q

63) An elderly woman falls down the stairs and suffers a Malgaigne fracture of the pelvis and a 1% degloving injury to the left forearm. Due to significant medical co-morbidity, the decision is made not to treat with surgery. The patient dies overnight on the ward. What is the most likely mechanism of death?

a. pulmonary embolism

b. fat embolism

c. septicaemia

d. myocardial infarction

e. intra-abdominal haemorrhage

A

e. intra-abdominal haemorrhage

A Malgaigne fracture of the pelvis is a fracture of the ischiopubic rami and an ipsilateral sacroiliac joint (or para-articular) fracture, and occurs due to high-energy blunt trauma.

It represents complete disruption of the pelvic ring and therefore an unstable fracture.

In such fractures, distortion and disruption of the pelvic soft tissues and vascular injury involving the rich blood supply in the pelvis will not be tamponaded by the bony ring, as the pelvis will expand to accommodate ever-increasing haematoma.

Mortality rate from major pelvic trauma is 10%; other common causes of death include multiorgan failure and sepsis, the latter expected to take several days to evolve.

How well did you know this?
1
Not at all
2
3
4
5
Perfectly
17
Q

70) Which of the following local factors is not associated with an increased risk of fracture non-union?

a. infection

b. fracture mobility

c. avascular fragments

d. impaction

e. open fracture

A

d. impaction

Fracture sites that have a poor blood supply, either as a result of the original injury or due to subsequent surgical treatment, may go on to develop atrophic non-union where the bone ends become osteoporotic, thin and pointed (osteolysis) with no evidence of fracture healing.

A fracture site that is very mobile may develop hypertrophic non-union where there is attempted healing denoted by excessive callus formation, but the fracture cleft persists.

Open fractures are often high energy, with soft-tissue damage and comminution of fracture fragments, and are prone to infection, all of which predispose to non-union.

Osteomyelitis in any fracture can result in delayed, non- or malunion.

Non-union in most skeletal locations should not be diagnosed radiographically until 6 months have passed, particularly in the presence of complicating factors.

How well did you know this?
1
Not at all
2
3
4
5
Perfectly
18
Q

74) A middle-aged man has a history of an undiagnosed wrist injury interfering with his playing golf. He presents with clinically apparent ulnar nerve compression at the wrist. Which of the following causes is most likely to be identified following investigation with CT and MRI?

a. non-union of hook of hamate fracture

b. non-union of scaphoid wrist fracture

c. scapholunate dissociation

d. pisiform osteoarthritis

e. triangular fibrocartilage complex tear

A

a. non-union of hook of hamate fracture

Fractures of the hook of the hamate are the most frequent type of hamate fracture, and most often occur from the repetitive stress of swinging a bat, club or racket, or from the direct blow of a club on the ground. This may result in ulnar nerve compression at the wrist in Guyon’s canal, which is particularly exacerbated in the context of non-union due to secondary osteoarthritis or loose bodies in the pisotriquetral joint.

Other causes of ulnar nerve compression at the wrist include adjacent masses, anomalous muscles and tendons, fibrous palmar arch, ulnar artery aneurysm, primary osteoarthritis of the pisotriquetral joint, os hamuli proprium and dislocation of the pisiform bone.

How well did you know this?
1
Not at all
2
3
4
5
Perfectly
19
Q

75) A 16-year-old female gymnast sustains a twisting injury to the knee, which becomes immediately painful and swollen, and she is unable to bear weight. Initial radiographs show an effusion but are otherwise normal. MRI confirms a joint effusion with a torn medial retinaculum, marrow oedema affecting the anterior aspect of the lateral femoral condyle, and a chondral defect of the medial facet of the patella. What is the most likely injury?

a. lateral collateral ligament tear

b. medial meniscal tear

c. pivot shift injury

d. transient patellar dislocation

e. posterolateral corner syndrome

A

d. transient patellar dislocation

Transient patellar dislocation always occurs laterally and was originally thought to be an injury confined to teenage girls with abnormal patellofemoral anatomy, but is now considered a potential injury in anyone who partakes in athletic activity.

The most common finding is effusion and lateralization of the patella with or without an abnormally shallow femoral sulcus.

Other findings seen on MRI are contusions of the lateral femoral condyle and medial patella with potential osteochondral defects, and disruption or sprain of the medial retinaculum.

Less specific findings include loose bodies and associated ligamentous or meniscal injury.

How well did you know this?
1
Not at all
2
3
4
5
Perfectly
20
Q

3 A 78 year old woman has left hip pain following a fall. On examination the left hip is shortened and externally rotated. X-rays show a fracture of the left neck of femur. Which of the following fractures is at highest risk of avascular necrosis?

(a) Basi-cervical fracture

(b) lnter-trochanteric fracture

(c) Sub-capital fracture

(d) Sub-trochanteric fracture

(e) Trans-cervical fracture

A

(c) Sub-capital fracture

Proximal femoral fractures are divided in subcapsular and extracapsular.

The joint capsule runs from the acetabulum to the inter-trochanteric line anteriorly and to the junction of the middle/ distal third of the femoral neck posteriorly.

Thus intracapsular fractures are those of the femoral neck: sub-capital, trans-cervical and basicervical fractures,

and extracapsular include inter-trochanteric, subtrochanteric and femoral shaft fractures.

lntracapsular fractures are more susceptible to AVN because the main supply to the femoral head is from the circumflex femoral arteries, which enter via the capsule and are more likely to be disrupted by such fractures.

Additional blood supply from the ligamentum teres artery via the acetabular fossa and the retinacular branches on the surface of the femoral neck, are usually insufficient to prevent AVN.

Furthermore they may be compromised by the raised pressure secondary to blood within the joint capsule.

The more proximal the subcapsular fracture, greater risk of vascular compromise, thus a sub-capital fracture has greatest risk of AVN.

How well did you know this?
1
Not at all
2
3
4
5
Perfectly
21
Q

(MSK) 22 With regards to the Ottawa rules for plain radiographs of the foot and ankle, which of the following is incorrect?

(a) An ankle X-ray is indicated if the patient cannot weight bear at the time of injury

(b) An ankle X-ray is indicated if there is pain in the malleolar zone and bony tenderness over the base of the 5th metatarsal

(c) A foot X-ray is indicated if there is pain in the midfoot zone and bony tenderness over the navicular

(d) An ankle X-ray is indicated if the patient cannot weight bear in the department

(e) An ankle X-ray is indicated if there is pain in the malleolar zone and bony tenderness over the posterior edge of the lateral malleolus

A

(b) An ankle X-ray is indicated if there is pain in the malleolar zone and bony tenderness over the base of the 5th metatarsal

A foot x-ray is indicated with bony tenderness over the base of the 5th metatarsal (and midfoot pain).

How well did you know this?
1
Not at all
2
3
4
5
Perfectly
22
Q

@# 24 A 32 year old man falls on his outstretched right wrist whilst playing football. Wrist X-ray reveals a displaced, oblique intraarticular fracture of the dorsal lip of the distal radius. What is the fracture type described?

(a) Barton’s

(b) Chauffeur’s

(c) Calles’

(d) Smith’s

(e) Salter Harris Type II

A

(a) Barton’s

A Barton’s fracture refers to an intra-articular fracture through the distal radius. The conventional Barton fracture involves the dorsal rim of the radius, the reverse Barton involves the volar rim.

Salter-Hanis fractures involve the growth plate before closure;

the other types described are not intra-articular.

Calles’ and Smith’s are fractures of the distal radius with dorsal and volar displacement of the distal fragments, respectively.

A chauffeur fracture is a triangular fracture of the radial styloid process.

How well did you know this?
1
Not at all
2
3
4
5
Perfectly
23
Q

29 A man suffers a supination-abduction ankle injury. Plain films reveal an oblique fibular fracture through the tibiofibular syndesmosis. Which Weber category does this represent?

(a) A

(b) B

(c) C

(d) D

(e) E

A

(b) B

The Weber classification is based on the location of the distal fibular fracture relative to the tibiotalar joint.

Type A is a transverse fracture distal to the ankle joint;

type B is an oblique fracture at the level of the joint;

type C is a fibular fracture proximal to the level of the joint.

There is no type D or E.

How well did you know this?
1
Not at all
2
3
4
5
Perfectly
24
Q

33 A patient has injured his knee and is unable to weight bear. AP and lateral radiographs are taken. Which of the following is an abnormal finding?

(a) On the lateral view, the distance from the lower pole of the , patella to the tibial tubercle is 1.5 times the length of the patella

(b) There is irregularity of the tibial tubercle

(c) On the AP view, a perpendicular line drawn from the lateral margin of the femoral condyle has 3 mm of the lateral margin of the tibal condyle outside of it

(d) There is a corticated, calcified body in the lateral head of the gastrocnemius muscle

(e) There is a multipartite patella

A

(a) On the lateral view, the distance from the lower pole of the , patella to the tibial tubercle is 1.5 times the length of the patella

On the lateral view, the distance from the lower pole of the patella to the tibial tubercle should equal the length of the patella plus or minus 20%. If this rule is broken, a ruptured patellar ligament must be suspected. On the AP view a perpendicular line drawn from the lateral margin of the femoral condyle should have s; 5 mm of the lateral margin of the tibal condyle outside of it. The fabella is a common sesamoid bone within the lateral head of gastrocnemius.

How well did you know this?
1
Not at all
2
3
4
5
Perfectly
25
Q

34 A patient presents with foot and ankle tenderness after a sports injury. Plain radiographs of the ankle and foot are taken. Which of the following is an abnormal finding?

(a) Bohler’s angle is 31 degrees

(b) On an AP view of the midfoot, the medial margin of the second metatarsal aligns with the medial margin of the intermediate cuneiform

(c) On an AP view of the midfoot, the medial margin of the third metatarsal aligns with the medial margin of the lateral cuneiform

(d) The width of the space between the distal tibia and fibula at a point 1 cm from the articular surface is 8 mm

(e) On the AP view of the midfoot, there is a lucent line through the base of the 5th metatarsal which runs parallel to the metatarsal shaft

A

(d) The width of the space between the distal tibia and fibula at a point 1 cm from the articular surface is 8 mm

The width of the space between the distal tibia and fibula at a point 1 cm from the articular surface should be s; 6 mm.

Bohler’s angle is normally 30-40 degrees.

In contradistinction to a fracture, the long axis of an unfused apophysis of the base of the 5th metatarsal runs parallel to the metatarsal shaft.

On an AP view of the midfoot, the medial margin of the second metatarsal should align with the medial margin of the intermediate cuneiform.

On an AP view of the midfoot, the medial margin of the third metatarsal aligns with the medial margin of the lateral cuneiform.

How well did you know this?
1
Not at all
2
3
4
5
Perfectly
26
Q

@# 36 A patient has injured his right shoulder. An AP view demonstrates an acromio-clavicular distance of 12 mm and a coraco-clavicular distance of 10 mm. The clavicle is not otherwise grossly displaced. What is the grade of the acromio-clavicular joint injury?

(a) Grade I

(b) Grade II

(c) Grade Ill

(d) Grade IV

(e) Grade V

A

(b) Grade II

How well did you know this?
1
Not at all
2
3
4
5
Perfectly
27
Q

38 A patient presents with tenderness in the anatomical snuffbox and a scaphoid series of plain radiographs are taken. There is a fracture across the proximal pole of the scaphoid. Which of the following is incorrect?

(a) Compared to other scaphoid fractures, those across the proximal pole have the highest risk of avascular necrosis

(b) Most scaphoid fractures occur across the waist

(c) Most scaphoid fractures are not displaced

(d) Scaphoid waist fractures may take up to 2 years to heal

(e) A vertical oblique fracture is considered more stable compared to a transverse fracture

A

(e) A vertical oblique fracture is considered more stable compared to a transverse fracture

Fractures across the proximal pole and waist both carry a high risk of subsequent AVN. 80% of fractures occur across the waist, compared to 10% for each of the poles. Transverse/horizontal oblique fractures are relatively stable compared to vertical oblique fractures.

How well did you know this?
1
Not at all
2
3
4
5
Perfectly
28
Q

40 A patient presents with suspected transient patellar dislocation. MR imaging is performed. Which of the following MR imaging features would be least expected in this condition?

(a) Disruption of the medial retinaculum

(b) Lateral patellar tilt

(c) Elevation of the vastus medialis obliquus muscle

(d) Bone contusion of the medial aspect of the medial femoral condyle

(e) Bone contusion of the inferomedial aspect of the patella

A

(d) Bone contusion of the medial aspect of the medial femoral condyle

In transient patella dislocation, the patella dislocates laterally and then relocates. This causes impaction between the inferomedial aspect of the patella and the anterolateral aspect of the lateral femoral condyle, which results in bone contusions. In addition to the above signs, a haemarthrosis is also usually present.

How well did you know this?
1
Not at all
2
3
4
5
Perfectly
29
Q

45 An MR arthrogram of the shoulder is performed in a patient with a known history of shoulder dislocation. Which of the following features would be more supportive of prior posterior, rather than anterior, glenohumeral dislocation?

(a) Hill-Sach’s lesion

(b) Anterior labral tear

(c) Torn glenohumeral ligaments

(d) Posterior capsule stripping

(e) Ban kart lesion

A

(d) Posterior capsule stripping

Posterior dislocations produce posterior, rather than anterior, capsule stripping. Other evidence of a prior posterior dislocation include: a reverse Bankart lesion, a reverse Hill-Sach’s lesions, or a posterior labral tear.

How well did you know this?
1
Not at all
2
3
4
5
Perfectly
30
Q

@# 53 A 15 year old boy presents with a history of right knee clicking, locking and intermittent swelling. There is no clear history of trauma. MRI shows a focus of abnormal signal in the subarticular marrow, a defect in the overlying cartilage, a loose intra-articular body and a small effusion.
What is the likeliest site of the cartilaginous defect?

(a) Medial aspect of the lateral condyle

(b) Lateral aspect of the lateral condyle

(c) Medial aspect of the medial condyle

(d) Lateral aspect of the medial condyle

(e) Posterior surface of the patella

A

(d) Lateral aspect of the medial condyle

Osteochondritis dissecans is the fragmentation and sometimes separation of a portion of the articular surface usually seen in adolescent males. It is most commonly related to repetitive
microtrauma although associations with other conditions such as Osgood-Schlatter’s and Scheuerman’s disease have been reported. It is most common in the medial femoral condyle although the humerus, capitellum and talus may be involved. It is bilateral in 10-20%.

How well did you know this?
1
Not at all
2
3
4
5
Perfectly
31
Q

7 A 20 year old man presents after a fall with anatomical snuff box tenderness. Which of the following statements is incorrect?

(a) 40% of scaphoid fractures are visible on initial radiographs

(b) Interval radiographs at 7-10 days detect the majority of initial occult fractures

(c) The MRI sequences of choice are coronal T1 and STIR

(d) 80% of fractures occur through the waist of the scaphoid

(e) Distal radius fractures can present in this manner

A

(a) 40% of scaphoid fractures are visible on initial radiographs

85% of scaphoid fractures are detectable initially using scaphoid views. Repeat views are commonly obtained but detection of occult fractures are unreliable. Nuclear imaging is sensitive but non-specific. MRI is the best test for detection of occult fracture. However, lack of availability means that other tests are still used.

How well did you know this?
1
Not at all
2
3
4
5
Perfectly
32
Q

@# 17 A young man avulses his anterior inferior iliac spine whilst kicking a football. The origin of which muscle will be affected?

(a) Adductor magnus

(b) lliopsoas

(c) Rectus femoris

(d) Sartorius

(e) Tensor fasciae latae

A

(c) Rectus femoris

The anterior inferior iliac spine is the origin of rectus femoris.

How well did you know this?
1
Not at all
2
3
4
5
Perfectly
33
Q

@#1 18 A man falls from a balcony and is brought into A&E. There is clinical shortening of the left leg and plain radiographs demonstrate fractures through the left sacroiliac joint and the left pubic rami. What is the most appropriate description of this fracture?

(a) Bucket-handle

(b) Duverney

(c) Open-book

(d) Malgaigne

(e) Wide-swept pelvis

A

(d) Malgaigne

A Malgaigne fracture typically involves two fractures, both on the same side of the pelvic ring: one anterior to the acetabulum (e.g. both pubic rami) and one posterior to the acetabulum (e.g. through the ilium, or the SI joint).

How well did you know this?
1
Not at all
2
3
4
5
Perfectly
34
Q

22 A 35 year old man involved in an RTA falls from a motorcycle and injures his forearm. Plain radiographs show a comminuted displaced radial head fracture with dislocation of the radioulnar joint. What is the eponymous term for this injury?

(a) Monteggia fracture

(b) Reverse Monteggia fracture

(c) Essex-Lopresti fracture

(d) Galeazzi fracture

(e) Maisonneuve fracture

A

(c) Essex-Lopresti fracture

A Galeazzi fracture is radial shaft fracture with subluxation/ dislocation of the distal radioulnar joint. A Monteggia fracture is a fracture of the ulnar shaft with dislocation of the radial head. The reverse version refers to the angulation/ displacement of the ulnar fracture and radial head dislocation. A Maisonneuve fracture refers to a fracture of the upper third of the fibula with a tear of the distal tibiofibular syndesmosis.

How well did you know this?
1
Not at all
2
3
4
5
Perfectly
35
Q

(Ped) 22 An active 13 year old boy complains of acute pain in the right hip following a game of football. An AP pelvis x-ray shows an irregular bone fragment below the right inferior pubjc ramus. What muscle has been avulsed?

(a) Adductor magnus

(b) lliopsoas

(c) Pectineus

(d) Rectus femoris

(e) Vastus medialis

A

22 (a)

Avulsion injuries to the pelvis are common in active adolescents, injury results from sudden traction of a muscle on an apophysis. The commonest is the adductor/ hamstrings insertion onto the ischial tuberosity of the inferior pubic ramus. Other pelvic avulsions include ASIS (sartorius muscle), AllS (rectus femoris) and the lesser trochanter (iliopsoas). Pectineus originates from the superior pubic ramus.

How well did you know this?
1
Not at all
2
3
4
5
Perfectly
36
Q

(Ped) 24 Which of the following eponymous osteochondroses, common in the paediatric population, is incorrectly described?

(a) Blount - proximal medial tibial epiphysis

(b) Frieberg’s - metatarsal head

(c) Kohler’s - scaphoid

(d) Perthe’s - femoral head

(e) Sever’s - calcaneal apophysis

A

(c) Kohler’s - scaphoid

Kohler’s disease affects the navicular bone, Preiser’s disease is osteochondrosis of the scaphoid. Other common paediatric osteochondroses include: Panner’s disease (humeral head), OsgoodSchlatter disease (tibial tubercle), and Scheuermann’s disease (vertebral apophysis).

How well did you know this?
1
Not at all
2
3
4
5
Perfectly
37
Q

32 A 32 year old workman presents with wrist pain. Plain radiographs show sclerosis and deformity of the lunate.
Which of the following statements is incorrect?

(a) This condition is more common on the right

(b) There is an association with positive ulnar variance

(c) Despite the presence of pain, the radiographs can initially be normal

(d) Scapholunate separation is a complication

(e) It is more commonly bilateral

A

(b) There is an association with positive ulnar variance

AVN of the lunate (Kienbock’s disease) is associated with negative ulnar variance in 75%. Manual labour is a predisposing factor. Other complications include ulnar deviation of the triquetrum and degenerative joint disease in the radiocarpal and midcarpal compartments

How well did you know this?
1
Not at all
2
3
4
5
Perfectly
38
Q

50 An epileptic patient complains of left shoulder pain after a seizure. A posterior dislocation is suspected. Which part of the humeral head is at risk of a compression fracture?

(a) Anteromedial

(b) Anterolateral

(c) Posteromedial

(d) Posterolateral

(e) Surgical neck

A

(a) Anteromedial

This is the ‘trough’ sign also known as a ‘reverse Hill-Sachs’ lesion and is caused by contact against the posterior glenoid labrum during posterior dislocation.

How well did you know this?
1
Not at all
2
3
4
5
Perfectly
39
Q

@# 51 A patient presents with wrist pain subsequent to a fall. Carpal instability is suspected and a lateral radiograph is taken of the wrist in neutral alignment. The scapholunate angle is 70° and the capitolunate angle is 10°. What is the most appropriate description?

(a) Normal

(b) Scapholunate dissociation

(c) Volar intercalated segment instability

(d) Dorsal intercalated segment instability

(e) Scapholunate advanced collapse

A

(b) Scapholunate dissociation

How well did you know this?
1
Not at all
2
3
4
5
Perfectly
40
Q

60 A 30 year old man is hit by a car sustaining a ‘bumper’ type injury. Radiographs show a bicondylar tibial plateau fracture with an ‘inverted Y’ appearance. In the Schatzker classification, which type of fracture is this?

(a) I

(b) II

(c) III

(d) IV

(e) V

A

(e) V

Type I (6%) is a pure wedge shaped cleavage fracture. Type II (25%) is a mixed cleavage and median compression fracture. Type Ill (36%) is a pure compression (depression) fracture. Type IV (10%) is a medial comminuted plateau fracture. Type VI (20%) is a transverse/oblique fracture separating the metaphysis from the diaphysis. Type V fractures are the rarest.

How well did you know this?
1
Not at all
2
3
4
5
Perfectly
41
Q
  1. In a 65 year old woman with a fracture of the neck of the humerus, which of the following classification systems to describe the fracture would be useful in guiding the surgical management?

a. Garden classification

b. Neer classification

c. Weber classification

d. Fryman system

e. Crosby–Fitzgibbon system

A
  1. b. Neer classification

The Neer classification system is used to grade humeral neck fractures. This system describes four parts – greater tuberosity, lesser tuberosity, humeral head and shaft ofhumerus. According to Neer, a fracture is displaced if there is more than 1 cm of displacement and 45_ angulation between any two segments. Two-part fractures involve any of the four parts and include one fragment that is displaced. Three-part fractures include a displaced fracture of the surgical neck in addition to either a displaced greater tuberosity or lesser tuberosity fracture. Four-part fractures include displaced fractures of the surgical neck and both tuberosities.

How well did you know this?
1
Not at all
2
3
4
5
Perfectly
42
Q
  1. A 45 year old woman falls onto her outstretched hand. The following findings on PA and lateral wrist plain films indicate which pathology? A scapholunate angle of 70, a capitolunate angle of less than 20, and a 4mm gap between scaphoid and lunate on PA view.

a. Normal appearances

b. Scapholunate dissociation

c. Volar intercalated segment instability (VISI)

d. Dorsal intercalated segment instability (DISI)

e. Perilunate dislocation

A
  1. b. Scapholunate dissociation

In scapholunate dissociation the scapholunate angle is >60_ and there is a >3mm gap between the scaphoid and lunate on AP view of the wrist. In VISI, capitolunate angle is increased and there is volar angulation of the lunate. In DISI, both scapholunate and capitolunate angles are increased and there is dorsal angulation of the lunate.

How well did you know this?
1
Not at all
2
3
4
5
Perfectly
43
Q
  1. A young man presents to A&E following a fall onto his outstretched right arm. Plain films of the right forearm show a fracture of the distal forearm with volar angulation of the distal fragment with no intra-articular component. The carpal bones remain well aligned. Which of the following injuries has he sustained?

a. Smith’s fracture

b. Barton’s fracture

c. Monteggia fracture

d. Galeazzi fracture

e. Colles fracture

A
  1. a. Smith’s fracture

This description is of a Smith’s fracture. More common is a Colles fracture, which is a fracture of the distal radius with dorsal angulation of the distal fragment. A Monteggia fracture is fracture of the ulnar with dislocation of the radial head. A Galeazzi fracture is a fracture of the radius with dislocation of the distal ulnar. Barton’s fracture is a fracture of the distal radius with dislocation of the distal radiocarpal joint.

How well did you know this?
1
Not at all
2
3
4
5
Perfectly
44
Q
  1. A 32 year old footballer sustains an avulsion injury to the anterior superior iliac spine during training. Which of the following muscles is likely to be affected?

a. Sartorius

b. Gracilis

c. Iliopsoas

d. Rectus femoris

e. Semimembranosus

A
  1. a. Sartorius

Sartorius has its origin at the anterior superior iliac spine and inserts into the pes anserinus. A sartorius muscle injury can therefore cause an avulsion fracture of the anterior superior iliac spine. Gracilis has its origin at the inferior pubic ramus, and rectus femoris has its origin at the anterior inferior iliac spine.

How well did you know this?
1
Not at all
2
3
4
5
Perfectly
45
Q
  1. A 21 year old long-distance runner complains of increasing right groin pain. Plain films show no acute bony injury, but demonstrate a pistol grip deformity of the femoral head, an osseous bump deforming the femoral head–neck junction and an alpha angle of 70_. The acetabulum appears normal. The most likely diagnosis is:

a. Hip dysplasia

b. Pincer-type acetabular impingement

c. Cam-type acetabular impingement

d. Sportsman’s hernia

e. Avascular necrosis

A
  1. c. Cam-type acetabular impingement

Femoroacetabular impingement (FAI) occurs as a result of repetitive microtrauma due to an anatomic conflict between the proximal femur and the acetabular rim at the extremes of motion. An osseous bump at the femoral head–neck junction is present in 50% of cam-type FAI and only 33% of pincer-type FAI. An alpha angle of >55_ is indicative of cam-type FAI. The alpha angle, drawn on the AP pelvis radiograph, is formed by a line drawn from the centre of the femoral head through the centre of the femoral neck, and a line from the centre of the femoral head to the femoral head–neck junction, found by the point by which the femoral neck diverges from a circle drawn around the femoral head. A normal patient’salpha angle is around 45, whereas for patients with FAI it may be around 70.

How well did you know this?
1
Not at all
2
3
4
5
Perfectly
46
Q
  1. A 24 year old rugby player attends A&E following a tackle during which he felt his left shoulder dislocate. Initial plain radiographs confirm an anterior inferior dislocation of the left shoulder. Which of the following statements is true?

a. The humeral head lies inferior and lateral to the glenoid on the AP view

b. The presence of a Hill–Sachs defect indicates previous dislocation

c. Hill–Sachs lesions are more common than Bankart lesions

d. Anterior dislocation accounts for 50% of shoulder dislocations

e. A Hill–Sachs lesion affects the inferior aspect of the humeral head

A
  1. c. Hill–Sachs lesions are more common than Bankart lesions

A Hill–Sachs lesion affects the postero-superior aspect of the humeral head and whilst it does often indicate a previous dislocation, this is not necessarily the case and it can be present after a single episode. A Bankart lesion affects the inferior glenoid. Almost 95% of all shoulder dislocations are anterior.

How well did you know this?
1
Not at all
2
3
4
5
Perfectly
47
Q
  1. Following a traumatic left elbow fracture, a young man complains of paraesthesia in his left ring and little fingers. He also starts to notice weakness of his left hand. A diagnosis of ulnar nerve entrapment is made. Which of the following muscles will not be affected?

a. Abductor digiti minimi

b. Abductor pollicis brevis

c. Adductor pollicis

d. Flexor carpi ulnaris

e. Flexor digiti minimi

A
  1. b. Abductor pollicis brevis

Abductor pollicis brevis is supplied by the median nerve and would therefore not be affected in an ulnar nerve injury. Due to the anatomic location of the ulnar nerve at the elbow, it can often be damaged leading to denervation and paralysis of the muscles supplied by the nerve. This includes the intrinsic muscles of the hand, which can be very debilitating. Injury to the ulnar nerve at the wrist would lead to severe muscle denervation sparing only the opponens pollicis, the superficial head of the flexor pollicis brevis and the lateral two lumbricals.

How well did you know this?
1
Not at all
2
3
4
5
Perfectly
48
Q
  1. A 56 year old woman slips off the pavement onto the road and her outstretched foot is run over by a passing car. She has immediate severe midfoot pain. Plain radiographs taken on arrival at the emergency department confirm a Lisfranc fracture dislocation of the midfoot. Which two bones does the Lisfranc ligament attach to?

a. First metatarsal and intermediate cuneiform

b. First metatarsal and medial cuneiform

c. Second metatarsal and medial cuneiform

d. Second metatarsal and intermediate cuneiform

e. First and second metatarsals to the medial and intermediate cuneiforms

A
  1. c. Second metatarsal and medial cuneiform

The Lisfranc ligament attaches between the second metatarsal and medial cuneiform, which is why an injury to this ligament allows the second to fifth metatarsals to drift laterally once they have lost this stabilisation. This is therefore an unstable injury and requires rapid immobilisation. This is a vital injury to detect as long-term sequelae will often result from a delayed diagnosis.

How well did you know this?
1
Not at all
2
3
4
5
Perfectly
49
Q

(Ped) 37. A seven year old boy presents to the minor injuries unit following a minor fall five weeks earlier. He complains of pain around the left elbow joint and has a limited range of movement. Blood tests are normal. Plain film of the elbow shows a small joint effusion and fragmentation of the capitellar epiphysis. Which one of the following is the most likely diagnosis?

a. Osteochondrosis of the capitellum (Panner’s disease)

b. Osteomyelitis

c. Osteochondritis dessicans

d. Juvenile chronic arthritis

e. Osteochondral capitellum fracture

A
  1. a. Osteochondrosis of the capitellum (Panner’s disease)

The most likely cause is osteochondritis of the capitellum. The blood supply to the capitellum is relatively fragile and osteochondritis usually occurs following a minor injury. The condition usually resolves spontaneously with no long-term complications. Osteochondritis dessicans tends to occur in adolescents and often produces a loose body within the joint causing symptoms of locking.

How well did you know this?
1
Not at all
2
3
4
5
Perfectly
50
Q
  1. Regarding scaphoid fractures, which of the following statements is correct?

a. 80% of scaphoid fractures occur at the waist

b. Approximately 5% of scaphoid fractures are complicated by avascular necrosis

c. Injury is typically due to hyperextension

d. Up to 60% of scaphoid fractures cannot be seen on initial radiograph

e. The specificity of CT in diagnosing scaphoid fractures is 60–70%

A
  1. c. Injury is typically due to hyperextension

The scaphoid bone is the most commonly fractured carpal bone and the mechanism is usually a fall onto the outstretched hand – ie. hyperextension of the wrist. The reported sensitivities and specificities of CT are 89–97% and 85–100%, respectively. The high negative predictive value of CT (96.8–99%) makes it very useful for ruling out a fracture. Scaphoid fractures are missed on initial radiographs in up to 30% of cases.

51
Q
  1. A 24 year old man has injured his right ankle playing football. The A&E SHO has asked your opinion on the plain radiographs. These show a widening of the medial joint space on the AP ankle view but no evidence of F, and an oblique fracture of the proximal shaft of the fibula. This is typical appearance for which of following F.?

a. Weber B

b. Maisonneuve

c. Pilon

d. Dupuytren’s

e. Fibula stress fracture

A
  1. b. Maisonneuve

This is the description of a Maisonneuve fracture (sometimes classified as Weber C3). This injury is often overlooked as the patient may complain only of ankle pain and hence a full tibia/fibula plain film is not taken. This fracture is often associated with ligamentous injury at the ankle, most usually of the anterior talofibular ligament and the postero-inferior talofibular ligament.

52
Q
  1. A 27 year old man falls onto his right hand during a game of rugby. He attends the A&E department and a plain film of the right hand shows a comminuted fracture through the base of the thumb metacarpal with an intra-articular component. This is the description of which of the following fractures?

a. Rolando’s fracture

b. Bennett’s fracture

c. Gamekeeper’s thumb

d. Boxer’s fracture

e. Barton’s fracture

A
  1. a. Rolando’s fracture

This is the classic description of a Rolando’s fracture. A Bennett’s fracture is also a fracture of the base of the first metacarpal but with no comminution; this fracture is often less stable than a Rolando’s fracture and more often requires surgical fixation. A ‘gamekeeper’s thumb’ often occurs as the result of forced abduction of the thumb and results in disruption of the ulnar collateral ligament.

53
Q

QUESTION 1
An athletic 19-year-old medical student presents to the Emergency Department after sustaining an injury to his right hip during training. A radiograph reveals a fracture of the anterior superior iliac spine. What is diagnosis?

A Avulsion of the adductor muscles

B Avulsion of the hamstring muscles

C Avulsion of iliopsoas

D Avulsion of rectus femoris

E Avulsion of sartorius

A

E Avulsion of sartorius

Avulsion injuries occur at characteristic sites and are particularly common in children and adolescents.

54
Q

A 60-year-old woman is assessed by the Emergency Department following a fall onto her right wrist. The initial radiograph shows an extra-articular fracture of the right distal radius, with volar subluxation of the distal fragment. Which eponymous fracture rype best matches this description?

A Barton’s fracture

B Colles’ fracture

C Hutchinson’s fracture

D Reverse Barton’s fracture

E Smith’s fracture

A

E Smith’s fracture

Although eponymous classifkation of injuries is often criticised, many eponyms endure as succinct descriptions of fracture patterns, forcing the radiologist to remain aware of the more common eponymous injuries.

55
Q

QUESTIONS
A 32-year-old builder is brought to the Emergency Department following a fall from scaffolding. He is believed to have fallen a considerable height and witnesses report that he landed on his feet. On primary survey, he is tachycardic, hypotensive and extremely tender on palpation of the pelvis and left hip. During resuscitation, a radiographic trauma series is obtained. What is the most likely pattern of pelvic injury?

A Bilateral fractures of the superior and inferior pubic rami

B Bilateral fractures of the superior and inferior pubic rami with a fracture through the left sacral ala

C Disruption of the sacroiliac joints and pubic symphysis

D Localised fracture through the left iliac wing

E Vertical fracture through the left ilium with fractures through the left superior and inferior pubic rami

A

E Vertical fracture through the left ilium with fractures through the left superior and inferior pubic rami

56
Q

QUESTION 10
A 14-year-old boy attends the Emergency Department following an injury to his right ankle in a rugby match. The radiograph shows a triplane fracture. Which fracture is likely to form part of this complex injury?

A Coronal fracture through the physis

B Coronal fracture through the epiphysis

C Horizontal fracture through the metaphysis.

D Sagittal fracture through the epiphysis

E Sagittal fracture through the metaphysis

A

D Sagittal fracture through the epiphysis

Triplane fractures are injuries of adolescence, occurring around the time of epiphyseal fusion. Partial fusion of the growth plate results in complex fracture geomeuy following injury; typically, this includes a coronal metaphyseal fracture, a horizontal fracture through the physis and a sagittal epiphyseal fracture (eg Salter-Harris IV inju1y). Tillaux fractures occur at a similar age, and are characterised by a horizontal fracture through the growth plate and a vertical epiphyseal fracture (Salter-Harris III injury).

57
Q

QUESTION 13
A 32-year-old man attends hospital following a fall onto his flexed left arm. He is referred to the duty orthopaedic team with a ‘Monteggia injuy’. What are the most likely radiological findings?

A A fracture of the distal radius with an associated dislocation of the radial head

B A fracture of the distal radius with an associated disruption of the distal radioulnar joint

C A fracture of the distal ulna with an associated dislocation of the radial head

D A fracture of the proximal ulna with an associated dislocation of the radial head

E A fracture of the proximal radius with an associated disruption of the distal radioulnar joint

A

D A fracture of the proximal ulna with an associated dislocation of the radial head

58
Q

A 25-year-old doctor injures her left wrist whilst snowboarding. Initial radiographs are reported as showing no fracture, but there is clinical suspicion of a scapholunate ligament disruption. Further views are obtained.
Which radiological feature would support the diagnosis?

A Scapholunate angle less than 30°

B Scapholunate distance of 2 mm

C ‘Signet ring’ appearance of the scaphoid

D Rotatoty subluxation of the lunate

E Wedge-shaped appearance of the lunate

A

C ‘Signet ring’ appearance of the scaphoid

The ‘signet ring’ appearance is due to rotatory subluxation of the scaphoid as a result of disruption of the scapholunate ligament.

59
Q

QUESTION 16
On an otherwise normal lateral radiograph of the knee, the patella is noted to be inferiorly situated, in keeping with patella baja. What is a possible association of this condition?

A Cerebral palsy

B Chondromalacia patella

C Juvenile idiopathic arthritis

D Quadriceps atrophy

E Recurrent patellar subluxation

A

C Juvenile idiopathic arthritis

On a lateral radiograph, the length of the patellar tendon should equal the height of the patella, plus or minus 20%. Patella baja is associated with polio, juvenile chronic arthritis and achondroplasia.

60
Q

QUESTION 20
A 50-year-old patient with right wrist pain and clicking is referred for a MRl by the orthopaedic surgeons, who suspect ulnolunate impingement. Which of the following imaging features would count against this provisional diagnosis?

A Disruption of the lunotriquetral ligament

B High signal within the lunate on T2w or STIR sequences

C Negative ulnar variance

D Perforation of the central portion of the triangular flbrocartilage (TFC)

E Sclerosis within the lunate

A

C Negative ulnar variance

61
Q

QUESTION 23
An 80-year-old woman is admitted to hospital following a fall. The patient had a right mastectomy and axillary dissection 5 years ago to treat an invasive ductal carcinoma. The pelvic radiograph reveals a left hip fracture. Which fracture site would be most suggestive of a pathological fracture?

A Greater trochanter fracture

B Intertrochanteric fracture of the left proximal femur

C Pertrochanteric fracture of the left proximal femur

D Subcapital fracture of the left neck of femur

E Subtrochanteric fracture of the left proximal femur

A

E Subtrochanteric fracture of the left proximal femur

An isolated lesser trochanter fracture is also highly suspicious for a pathological fracture.

62
Q

QUESTION 26
In a 40-year-old woman complaining of wrist pain, radiographs reveal sclerosis and collapse of the lunate, with rotato1y subluxation of the scaphoid. What is the most likely diagnosis?

A Freiberg’s disease

B Kienboeck’s disease

C Kohler’s disease

D Sever’s disease

E Sinding-Larsen disease

A

ANSWER: B

Kienboeck’s disease describes lunate collapse as a result of vascular insufficiency and avascular necrosis. Avascular necrosis of the navicular is Kohler’s disease whilst Freiberg’s disease describes avascular necrosis of the second metatarsal head. Sever’s disease relates to calcaneal apophysitis

63
Q

A 20-year-old footballer injured his ankle 3 months ago, but still experiences pain that prevents a return to competitive sport. An MRI of his injured ankle shows a partially detached osteochondral fragment within the lateral aspect of the talar dome, with fluid deep to the fragment and a fissure in the overerlying cartilage. Which stage of disease do these appearances represent?

A Stage 0

B Stage 1

C Stage 2

D Stage 3

E Stage 4

A

C Stage 2

64
Q

A young man with sickle cell disease presents to the Emergency Department with left hip pain. The orthopaedic team suspect avascular necrosis and request an MRI of both hips. The MRI shows a subchondral fracture of the left femoral head, with preservation of the joint space. The asymptomatic right hip is normal. How would the appearances of the left hip be classified?

A Stage 0 disease

B Stage I disease

C Stage II disease

D S rage III disease

E Stage IV disease

A

D S rage III disease

65
Q

QUESTION 39
A 35-year-old woman injures her right foot during a fall whilst wearinghigh-heeled shoes. She is subsequently unable to weightbear, and the entire foot is bruised, swollen and tender. A radiograph of the foot initially appears normal, but the possibility of a Lisfranc fracture-dislocation prompts close attention to the tarsometatarsal alignment. What is the normal alignment in this area?

A Fifth metatarsal aligned with the lateral cuneiform

B First metatarsal aligned with the cuboid

C Fourth metatarsal aligned with the medial cuneiform

D Second metatarsal aligned with the middle cuneiform

E Third metatarsal aligned with the navicular

A

D Second metatarsal aligned with the middle cuneiform

66
Q

(Ped) QUESTION40
A 15-year-old boy presents with a painful right knee which is aggravated by sport. It occasionally swells and locks, but clinical examination does not reveal any ligamentous instability. A plain radiograph appears normal and an MRI is performed and shows evidence of osteochondritis dissecans. Which one of the following statements is true regarding the imaging findings in this condition?

A Associated loose bodies are seen.

B Decreased signal within subchondral bone is seen on proton density MR images.

C MRI is of little use in predicting stability of fractures.

D The lateral condyle is most commonly involved.

E The plain radiograph is always normal.

A

A Associated loose bodies are seen.

Osteochondritis dissecans is a subchondral defect which may lead to separation of the subchondral bone.

67
Q

QUESTION 58
An 18-year-old man experiences persistent symptoms following a fracture through the waist of the right scaphoid. Radiographs of the right scaphoid indicate non-union. An MRl is performed to assess the vascularity of the proximal pole. Which imaging features are consistent with a diagnosis of avascular necrosis?

A Bone marrow enhancement following administration of gadolinium

B High signal surrounding the fracture on T2w images

C High signal within the proximal pole on Tl w images

D High signal within the proximal pole on STIR images

E Low signal within the proximal pole on Tl w images

A

E Low signal within the proximal pole on Tl w images

Low signal on Tl reflects death of the adipocytes. The combination of low signal on Tlw images and low or intermediate signal on T2w images accurately predicts avascular necrosis.

68
Q
  1. A 25-year-old male presents with a history of dislocation and spontaneous relocation of the patella while playing football. An MRI of the knee is requested. Which of the following findings is consistent with the clinical history of patellar dislocation?

A. Bone oedema involving medial facet of patella and medial femoral condyle.

B. Bone oedema involving posterior patella and anterior aspect of the tibial plateau.

C. Bone oedema involving the lateral facet of patella and lateral femoral condyle.

D. Bone oedema involving the lateral facet of patella and medial femoral condyle.

E. Bone oedema involving the medial facet of patella and lateral femoral condyle.

A
  1. E. Bone oedema involving the medial facet of patella and lateral femoral condyle.

Transient dislocation of the patella typically occurs laterally as a result of a twisting injury in a fixed and flexed knee. The medial facet of the dislocated patella impacts against the lateral femoral condyle, resulting in the classic bone contusion pattern. Rarely oedema may also be seen in the adductor tubercle of the medial femoral condyle due to avulsion of the medial patello-femoral ligament.

69
Q
  1. A 34-year-old female presents to the A&E department after falling on an outstretched hand. Examination reveals tenderness at the anatomic snuff box. A scaphoid radiograph series confirms scaphoid fracture. Which of the following features is most associated with a poor prognosis?

A. Fracture of the distal third.

B. Fracture of the middle third.

C. Fracture of the proximal third.

D. Horizontal oblique fracture orientation.

E. Displacement of the scaphoid fat stripe.

A
  1. C. Fracture of the proximal third.

Scaphoid fracture is the most common of all carpal bone fractures and also potentially serious due to the high rate of avascular necrosis. This fracture can be difficult to detect on initial radiographs. Wrist casting and repeat radiography after 1 week are typically advised if there is ongoing suspicion. Fractures of the proximal third account for 20% of injuries, but are associated with failure to unite in 90%. Middle third fractures make up the majority (70%), with up to 30% failing to reunite. Distal third fractures usually reunite. A vertical oblique fracture is more unstable than a horizontal oblique fracture. Fracture displacement of greater than 1mm is also a poor prognostic feature.

70
Q
  1. A 56-year-old man has a 6-week history of dull discomfort just above his right ankle. A plain ankle radiograph is performed and this demonstrates a relatively ill-defined area of lucency in the distal tibial metaphysis. An underlying aggressive lesion is suspected and the patient is referred for an MRI of the distal right leg. This shows a rather serpiginous-shaped lesion in the distal right tibia. A parallel rim of hypo- and hyperintensity is seen on one of the imaging sequences, which is very helpful in confirming that the lesion is secondary to metadiaphyseal osteonecrosis rather than a neoplasm. On which imaging sequence is this parallel rim most likely to be seen?

A. GE T2*WI.

B. Fast spin echo (SE) T1WI.

C. Fast SE T2WI.

D. Fast SE STIR.

E. Fast SE T1WI post gadolinium

A

13.C. Fast SE T2WI.

The parallel rim of hypo- and hyperintensity seen on T2WI refers to the ‘double line’ sign, which is almost pathognomic of osteonecrosis. It is most commonly associated with avascular necrosis of the femoral head, but can be seen in osteonecrosis at other sites on MRI. The ‘double line’ sign constitutes a hyperintense inner border (inflammatory response of bone with granulation tissue), with a hypointense periphery (reactive bone interface).
The characteristic plain radiographic pattern of metadiaphyseal osteonecrosis is that of a serpentine ring-like band of sclerosis that separates a central necrotic zone of variable lucency from surrounding normal marrow, although this pattern is a relatively late manifestation of osteonecrosis. Earlier in the course of the disease, osteonecrosis may result in a poorly defined region of lucency within the medullary space, a feature that may be indistinguishable from a lytic neoplastic process on x-ray. MRI is then very useful in these cases by showing the serpentine low signal rim of the lesion on T1WI. On T2WI, the rim of the lesion may have low signal, high signal or both (the ‘double line’ sign), the latter being the most specific sign for osteonecrosis.

71
Q
  1. A 39-year-old male presents with tenderness and decreased range of movement of the right elbow after falling on an outstretched arm while playing indoor football. A radial head fracture is noted on his radiographs, but the A&E doctor asks for your opinion, suspecting an additional injury. What is the most common associated fracture with this injury?

A. Olecranon fracture.

B. Coronoid process fracture.

C. Scaphoid fracture.

D. Proximal ulna fracture.

E. Capitellum fracture.

A
  1. B. Coronoid process fracture.

Radial head fractures are common, accounting for approximately one-third of all elbow fractures and up to 5% of all fractures in adults. A recent retrospective study found that associated fracture of the upper extremity was seen in 10.2% of patients, with fractures of the coronoid process the most common (4.1%). Radial head fracture, coronoid fracture, and medial collateral ligament tear form the ‘terrible triad’ of the elbow, which requires operative fixation.

72
Q
  1. A 21-year-old rugby player presents to the A&E department with right shoulder pain and decreased range of movement following a tackle. There is obvious contour deformity on examination. Plain radiographs confirm anterior dislocation. Which additional radiographic finding is in keeping with

a Hill–Sachs deformity?

A. Intra-articular loose body.

B. Greater tuberosity fracture.

C. Anterior glenoid rim fracture.

D. Anterior humeral head indentation.

E. Posterior humeral head indentation.

A
  1. E. Posterior humeral head indentation.

Anterior dislocation occurs when the arm is forcibly externally rotated and abducted. Radiographically, the humeral head lies inferior and medial to the glenoid on the AP view. The Hill–Sachs deformity is an indentation on the posterosuperior portion of the humeral head and indicates a greater likelihood of recurrence. A Bankart deformity is a bony fragment off the inferior glenoid. Anterior humeral head indentation is a ‘reverse Hill–Sachs’ deformity seen in posterior dislocations.

73
Q
  1. A patient attends A&E following an RTA in which she was the driver of car involved in a head-on collision. She complains of pain in both knees. Plain radiographs of the knees are unremarkable. Which of the following findings on MRI is most likely?

A. Bruising in the posterior aspect of the lateral tibial plateau and middle portion of the lateral femoral condyle.

B. Bruising at the anterior aspect of the tibia.

C. Kissing contusions in the anterior aspect of the distal femur and proximal tibia.

D. Bruising in the lateral femoral condyle with a second smaller area in the medial femoral condyle.

E. Bruising in the inferior medial patella and the anterior aspect of the lateral femoral condyle.

A
  1. B. Bruising at the anterior aspect of the tibia.

Such bruising occurs in a dashboard injury when a posteriorly directed force is applied to the anterior aspect of the proximal tibia with the knee in flexion, such as occurs in an RTA. Bruising is also occasionally found in the posterior patella in this situation. Associated soft-tissue injuries are disruption of the posterior capsule and posterior cruciate ligament (PCL). The pattern of injury in option A is caused by the pivot shift injury (valgus load in flexion combined with external rotation of the tibia or internal rotation of the femur). This will result in anterior cruciate ligament (ACL) disruption and the resultant anterior subluxation of the tibia causes impaction of the lateral femoral condyle against the posterolateral margin of the lateral tibial plateau. Soft-tissue injuries that may occur are tears of the posterior capsule, the posterior horn of the lateral or medial meniscus, and the medial collateral ligament (MCL). The kissing contusions in option C are as a result of hyperextension injury; resulting injuries may be to the ACL, PCL, or menisci. Option D describes the pattern found in clip injury, which involves a pure valgus stress while the knee is in mild flexion. The second area of bruising in the medial femoral condyle in this situation is due to avulsive stress to the MCL. The findings in option E are in keeping with transient lateral patellar dislocation, as discussed elsewhere in this chapter.

74
Q

(Ped) 22. A 12–year-old who is a keen athlete presents with left groin pain. A plain film of the pelvis reveals avulsion of the apophysis of the left ischial tuberosity. Which muscle attachment has he injured?

A. Sartorius.

B. Hamstrings.

C. Adductors.

D. Rectus femoris.

E. Iliopsoas.

A
  1. B. Hamstrings.

Avulsion fractures in the pelvis are generally uncommon injuries and are seen almost exclusively in adolescent athletes. They occur at the apophyses, which while growing are more prone to injury than the adjacent tendons. The hamstrings attach at the ischial tuberosity. Sartorius attaches at the anterior superior iliac spine, rectus femoris at the anterior inferior iliac spine, the adductors at the symphysis pubis, and iliopsoas at the lesser trochanter. Care should be taken not to mistake an old avulsion, which can produce irregularity, marked periostitis, and adjacent soft-tissue mineralization, for a more sinister lesion.

75
Q
  1. A 30-year-old female runner presents with a history of pain in the legs on running. Plain radiographs are unremarkable. An isotope bone scan reveals subtle, longitudinal, linear uptake on the delayed bone scan images, with normal angiogram and blood pool images. What is the diagnosis?

A. Stress fracture.

B. Shin splints.

C. Osteoid osteoma.

D. Osteomyelitis.

E. Hypertrophic osteoarthropathy

A
  1. B. Shin splints.

Excessive exertion of tibialis and soleus muscles of the legs causes periostitis along the muscular attachments. This results in longitudinal linear uptake on delayed bone scan images. The angiogram and blood pool images are usually normal compared to stress fracture, which is associated with hyperperfusion and hyperaemia. On delayed images focal fusiform uptake is seen with stress fracture. Infection is associated with hyperperfusion, hyperaemia, and focal increased uptake. Osteoid osteoma demonstrates hyperperfusion, hyperaemia, and focal double density due to nidus and reactive osteosclerosis. Paget’s disease is associated with increased uptake in an enlarged and deformed bone. Age and clinical presentation are also against this diagnosis. Hypertrophic osteoarthropathy is associated with irregular cortical uptake producing the ‘tramline’ sign.

76
Q
  1. A 26-year-old man presents to the A&E department with wrist pain and swelling after falling from a ladder on an outstretched hand. The lateral radiograph demonstrates posterior dislocation of the capitate relative to the lunate. What is the most commonly associated fracture with this injury?

A. Capitate.

B. Lunate.

C. Triquetral.

D. Scaphoid.

E. Radius.

A
  1. D. Scaphoid.

The findings describe perilunate dislocation, which is the most common carpal dislocation. It can occur without fracture (lesser arc injury) or with fracture (greater arc injury). Greater arc injuries are twice as frequent as lesser arc injuries. When describing these injuries the fracture is named first with the prefix ‘trans’ followed by the dislocation. Trans-scaphoid perilunate dislocation is the most common type of perilunate injury. Fractures of the trapezium, capitate, hamate, and triquetrum are also part of the greater arc injuries. Other radiographic signs of this injury include disruption of the carpal (Gilula) arcs and a triangular lunate on the AP view. An early sign is widening of the scaphoid–lunate space (Terry-Thomas sign), which suggests scapholunate dissociation. Lunate dislocation is the final stage of perilunate injuries, and is associated with the highest degree of instability.

77
Q
  1. A 34-year-old woman has chronic right wrist pain, with no documented history of previous trauma. An x-ray of the right wrist shows sclerosis and irregularity of the scaphoid with early bony fragmentation. What is the most likely eponymous disease that has resulted in this abnormality?

A. Sever disease.

B. Freiberg disease.

C. Kohler disease.

D. Iselin disease.

E. Preiser disease.

A
  1. E. Preiser disease.

The x-ray appearances are typical for osteonecrosis within the scaphoid. This is usually posttraumatic in aetiology, but when idiopathic it is known as Preiser disease. Postulated mechanisms for the osteonecrosis are repetitive minor trauma or secondary to drug treatment (e.g. steroids). The remaining wrong answers refer to osteochondroses affecting the foot. Freiberg disease affects the head of the second metatarsal, Kohler disease the tarsal navicular, Iselin disease the base of the fifth metatarsal and Sever disease the calcaneal apophysis.

78
Q
  1. A 76-year-old man presents with hip and pelvic pain. He has a past history of renal cell carcinoma treated by radiofrequency ablation, and has been treated on multiple occasions with heparin for thromboembolic disease. Plain films are non-contributory but a 99mTc bone scan reveals increased thoracic kyphosis and increased uptake in the body and bilateral alae of the sacrum in an H configuration. What is the most likely diagnosis?

A. Brown tumour.

B. Multiple myeloma.

C. Metastasis from renal cell carcinoma.

D. Chordoma.

E. Insufficiency fractures.

A
  1. E. Insufficiency fractures.

This patient has developed osteoporosis due to heparin administration (and with age). This has resulted in thoracic kyphosis and the ‘H’ sign of increased uptake within the sacral body and alae, which is classical of insufficiency fractures of the sacrum. Often there will have been relatively minor trauma that will not be reported by the patient, and there may be associated pubic rami fractures. Radiotherapy to the area (e.g. in gynaecological malignancy) is another predisposing factor Multiple myeloma, metastases from renal cell carcinoma, and chordoma are typically osteolytic and result in osteopenia at isotope bone scan (IBS), although the investigation has poor sensitivity for myeloma. Brown tumours do cause increased uptake on IBS, but we are not given a history of renal failure or hyperparathyroidism to explain their presence. There is no history given to suggest infection.
Bone metastases which cause an increased uptake on IBS are breast, prostate, lymphoma, pulmonary carcinoid, mucinous GI, and bladder tumours. Renal cell carcinoma, thyroid, and melanoma typically cause photopoenia.

79
Q

(Ped) 52. A 13-year-old boy presents with activity-related knee pain and locking. A plain film is unremarkable. An MRI reveals a defect in the lateral aspect of the medial femoral condyle. There is increased signal in the adjacent bone marrow on T2WI. What is the most likely diagnosis?

A. Distal femoral cortical irregularity.

B. Osteochondritis dissecans.

C. Sinding–Larsen–Johanssen disease.

D. Osgood–Schlatter disease.

E. Avulsion of the anterior tibial spine.

A
  1. B. Osteochondritis dissecans.

This disease is thought to represent a fracture secondary to avascular necrosis of an area of subchondral bone and cartilage, and peak incidence is at 12–13 years of age. Repetitive trauma, ischaemia, and familial tendency are common predisposing factors. In the knee, 75% of cases involve the postero-Lateral Aspect of the MEdial femoral condyle, hence the mnemonic LAME. The disease is bilateral in one-third of patients. A purely cartilaginous fragment will go unrecognized on plain film; a bony fragment and its donor site will not. Instability on MRI is suggested by fragments >1 cm, cysts larger than 5 mm within the donor site, high T2WI signal between the donor site and fragment (even if the cartilage is intact), and loose bodies within the joint. Distal femoral cortical irregularity (also referred to as cortical desmoid) is seen as a defect in the posterior aspect of the medial cortex of the distal femoral metaphysis, 1–2 cm proximal to the physis (at the attachment of adductor magnus). It appears low signal on T1WI and high signal on T2WI sequences. Plain film may show an ill-defined or scalloped appearance and may occasionally mimic an aggressive lesion (it is benign). Sinding–Larsen–Johanssen disease represents osteochondrosis at the site of attachment of the patellar tendon to the patella (inferior pole). Osgood–Schlatter disease represents apophysitis at the site of attachment of the patellar tendon to the tibia, namely the anterior tibial tubercle. Both are causes of knee pain and are associated with activity and repetitive traction microtrauma. An avulsion of the anterior tibial spine is the paediatric equivalent of an ACL tear (the chondroosseous tibial spine is the weakest part of the ACL complex prior to physeal closure). MRI reveals a low signal T1WI line through the anterior tibial spine.

80
Q
  1. A 55-year-old man is noted on a plain x-ray of pelvis to have a right hip prosthesis. There is a cemented acetabular component present with an uncemented stem. Of the following hip arthroplasties, which is the most likely procedure that he has undergone?

A. Unipolar hemiarthroplasty.

B. Bipolar hemiarthroplasty.

C. Hip resurfacing.

D. Hybrid total hip replacement.

E. Reverse hybrid total hip replacement.

A
  1. E. Reverse hybrid total hip replacement.

A combination of a cemented acetabular cup and an uncemented femoral stem is known as a reverse hybrid hip total hip replacement. A hybrid total hip replacement is a combination of a cemented femoral stem and an uncemented acetabular cup.
A unipolar hemiarthroplasty comprises a combination of a femoral component articulating directly with the native cartilage surface of the acetabulum. A bipolar hemiarthroplasty comprises a combination of a femoral component articulating with a cup inserted into the native acetabulum without fixation. This cup is usually made of polyethylene with a metal backing and can normally move within the native acetabular cavity as a result of the absence of fixation.
Hip resurfacing consists of replacing the surface of the femoral head by a metallic ‘cap’ without removing the femoral neck or instrumenting the femoral diaphysis. The cap used on the femoral head is virtually the same size as the natural head and articulates with an acetabular prosthetic cup, usually made of metal. This type of procedure is favoured in younger active patients and may allow for easier revision to a total hip replacement in later years

81
Q
  1. A 30-year-old male presents with a history of painful heels after a fall from a height. Plain radiograph demonstrates calcaneal fractures. Which of the following statements regarding calcaneal fractures is true?

A. Extraarticular fractures represent 75% of all calcaneal fractures.

B. Calcaneal fracture classification is based on fracture line location at the posterior facet.

C. Bilateral fractures are present in 30% of cases.

D. The flexor hallucis longus tendon passes inferior to the sustentaculum tali on the lateral aspect of the calcaneus.

E. Normal Boehler’s angle is less than 20°.

A
  1. B. Calcaneal fracture classification is based on fracture location at the posterior facet.

Calcaneal fractures represent 60% of fractures involving the tarsal bones. Axial loading resulting from a fall from a height is the most common cause followed by motor vehicle accidents. Treatment is based on accurate evaluation and classification of calcaneal fractures using multidetector CT reformats.
Calcaneal fractures are classified into intra-articular and extra-articular based on the involvement of the posterior facet of the subtalar joint. Intra-articular fractures, accounting for 75% of all calcaneal fractures, are further classified into four types depending on the number of fracture lines and fragments. Extra-articular fractures are classified into three types depending on whether the fracture involves the anterior, middle, or posterior aspect of calcaneus.
Bilateral fractures are seen in less than 10% of cases. Approximately 10% of calcaneal fractures are associated with compression injuries of the spine, commonly at the thoracolumbar junction. Boehler’s angle is formed by the intersection of (a) a line from the highest point of the posterior calcaneal tuberosity to the highest point of the posterior facet and (b) a line from the latter point to the highest of the anterior process. Normal Boehler’s angle is 20–40°. An angle less than 20° indicates collapse of the posterior facet. The sustentaculum tali is an eminence on the medial aspect of the calcaneus bearing the middle facet of subtalar joint.

82
Q

7 A bone scan is performed for the investigation of shin splints in a young athlete who complains of lower limb pain on running. Which option best describes the most likely finding in this diagnosis?

a Normal study

b Bilateral symmetrical uptake in the patellae (the `hot patella sign’)

c Linear symmetrical cortical uptake seen most avidly along the posterior aspects of the tibiae

d Linear asymmetrical uptake seen most avidly along the anterior aspects of the tibiae, the `leading leg’ being more avid

e Patchy uptake in the distal tibial shafts

A

7 Answer C: Linear symmetrical cortical uptake seen most avidly along the posterior aspects of the tibiae

Patchy uptake in shafts is more likely to indicate stress fracture, which may present in a similar fashion. The uptake in shin splints is usually diagnostic, but lateral views of the lower limbs are necessary to demonstrate the uptake along the posterior cortices. The hot patella sign is non-specific but not associated with the diagnosis of shin splints.

83
Q

(ped) 21 A 12-year-old boy presents with a three-month history of a painful knee, which clicks and locks. A PA radiograph of the knee shows a linear lucency separating a fragment of bone from the lateral aspect of the medial femoral condyle. MR shows the fragment to be of intermediate signal on all sequences. On T2-weighted imaging a rim of high signal separates this from the condyle. Which of the following is the most likely diagnosis?

a Spontaneous osteonecrosis

b Osteochondritis dissecans

C Blount disease

d Acute osteochondral fracture

e Osteogenesis imperfecta

A

21 Answer B: Osteochondritis dissecans

Osteochondritis dissecans is an osteochondral fracture caused by chronic injury. It most commonly affects the lateral aspect of the medial femoral condyle. Early in the disease, radiographs demonstrate either no abnormality or a joint effusion. In more advanced disease there is a completely detached, loose osteochondral fragment.

84
Q

22 A 16-year-old athlete presented with a history of locking of his knee following an injury. His plain radiograph demonstrates a small opacity within the joint space. What is the most likely diagnosis?

a Intra-articular loose body

b Calcium pyrophosphate deposition disease (pseudogout)

C Heterotopic ossification

d Anatomical variant

e Osteochondral fracture of the medial femoral condyle

A

22 Answer A: Intra-articular loose body

85
Q

25 A 20-year old man jumped 30 feet from a building onto concrete and landed on his feet. He was assessed in the Emergency Department and complained only of severe pain in his feel. He remained haemodynamically stable and after a full assessment it is clear that his only injuries are to his feet. What injury is he most likely to have sustained?

a Osteochondral fracture of talar dome

b March fractures of second and third metatarsals

C Comminuted calcaneal fractures

d Maisonneuve fracture

e Transverse fracture of fifth metatarsal

A

25 Answer C: Comminuted calcaneal fractures

Predictable patterns of injury after a fall from a height include: calcaneal, pelvic and thoraco-lumbar spine fractures. AMaisonneuve fracture is a proximal fibular fracture commonly associated with a medial malleolar avulsion fracture.

86
Q

27 An elderly lady fell onto her back and experienced severe back and buttock pain. Pelvic radiographs revealed marked osteopenia but no definite fracture. A bone scan was then performed, which showed an H-shaped pattern of increased uptake. Which bone is likely to be involved?

a Sacrum

b Ilium

C Ischium

d Pubis

e Fifth lumbar vertebra

A

27 Answer A: Sacrum

Osteoporotic sacral insufficiency fractures have a classical H-shaped configuration.

87
Q

28 A woman with poorly controlled diabetes mellitus fell down the stairs feet first. She did not experience much pain but found it difficult to walk after the fall and visited the Emergency Department where a doctor noticed that her ankle joint appeared abnormal with a large haematoma on the medial aspect of her foot. Neurological examination demonstrated bilateral sensory impairment in a stocking distribution. What injury is she most likely to have sustained?

a Osteochondral fracture of the talar dome

b Fracture of the medial malleolus

C Maisonneuve fracture

d Lisfranc fracture-dislocation

e Pilon fracture

A

28 Answer D: Lisfranc fracture-dislocation

A fracture of the first or second metatarsal in conjunction with a dislocation of the tarso-metatarsal articulation is termed a Lisfranc fracture-dislocation. Such injuries usually require a significant force. A fracture of the medial malleolus usually occurs as a result of an inversion injury. A fracture of the proximal fibula (with disruption of the proximal talofibular syndesmosis) in conjunction with a fracture of the medial malleolus is termed a Maisonneuve fracture. Diabetics are at risk of peripheral neuropathy and subsequent injury due to lack of sensation and the commonest cause of a Charcot joint in the Western world is diabetes

88
Q

29 A 44-year-old man had an accident in which he was thrown from a motorcycle. He landed on his left foot, which folded beneath him. On the day of the injury, his foot was examined and standard AP and lateral radiographs revealed no fracture. The foot was placed in slight plantar flexion and immobilised with a cast. After three days, the patient was re-evaluated and his foot remained oedematous. Ecchymoses was noted laterally and he could not weight-bear. What injury is he most likely to have sustained?

a No bony or ligamentous injury is likely

b Torn ligament between the second metatarsal and medial cuneiform

C Transverse fracture through base of fifth metatarsal

d Fracture of second metacarpal associated with dislocation of the tarsometatarsal joint

e Fracture of medial malleolus

A

29 Answer B: Torn ligament between the second metatarsal and medial cuneiform

An injury to the Lisfranc joint most commonly occurs at the joint involving the first and second metatarsals and the medial cuneiform. The Lisfranc ligament is the only ligament connecting the second metatarsal to the medial cuneiform and may be torn with a Lisfranc injury. While transverse ligaments connect the bases of the lateral four metatarsals, no transverse ligament exists between thefirst and second metatarsal bases. The joint capsule and Lisfranc ligament form the only minimal support on the dorsal surface of the Lisfranc joint. As many as 20% of Lisfranc joint injuries are missed on initial anteroposterior and oblique radiographs.

89
Q

33 A football goalkeeper dived to the ground while making a save and experienced immediate pain in his wrist. The action replay shows forced hyperextension in ulnar deviation. Radiographs reveal no abnormality on the AP projection, but a subtle fracture on the lateral view. What bone is most likely to have been injured?

a Capitate

b Hamate

c Lunate

d Pisiform

e Triquetral

A

33 Answer E: Triquetral

Triquetral fractures are common although easily missed on standard frontal projections. On a lateral projection an avulsed flake of bone lying posterior to the triquetral is typical. The usual mechanism of injury is falling onto an outstretched hand in ulnar deviation. A less common mechanism of triquetral fracture is a direct blow to the dorsum of the hand, which would usually be accompanied by other carpal fractures as a greater force is required. A fall onto an outstretched hand more commonly causes a scaphoid fracture although this is more likely in radial rather than ulnar deviation.

90
Q

34 A skier fell on his outstretched hand with his hand caught in his ski pole and sustained an abduction injury of his thumb. His thumb was painful and swollen and he noticed that it felt unstable. He consulted a doctor in the resort who arranged radiographs of his hand and wrist. What is the most likely diagnosis?

a Dislocation of carpo-metacarpal joint of thumb

b Soft-tissue injury only

C Scaphoid fracture

d Rupture of ulnar collateral ligament

e Dislocation of first metacarpophalangeal joint

A

34 Answer D: Rupture of ulnar collateral ligament

Chronic injury of this ligament is termed gamekeeper's thumb' but the acute form is now much more common and is termed skier’s thumb’ due to acute forceful abduction of the thumb usually when trapped in a ski pole. The ulnar collateral ligament originates from the first metacarpal head and may avulse a small fragment of bone from its proximal phalangeal insertion. Subluxation, but not usually frank dislocation, of this joint is seen and prompt consideration of surgical repair is indicated to prevent prolonged pain and functional impairment.

91
Q

2 A 17-year-old female presented with foot pain on walking. Plain films showed a well-defined cystic lesion in the second metatarsal head. What is a plain film six months later likely to show?

a Cortical thinning

b Sclerosis and flattening of the second metatarsal head

c Sclerosis of the other metatarsals

d Subluxation of the metatarsal heads

e Bony erosions in the metatarsal heads

A

2 Answer B: Sclerosis and flattening of the metatarsal head

The scenario described is of Freiberg disease, osteochondrosis of the head of the second (third and fourth) metatarsal heads in 10-18 year olds, usually women (M:F 1:3). Clinically they present with metatarsalgia, swelling and tenderness. Early and late features are seen. Early on, flattening and cystic lesions of the metatarsal head are seen, with widening of the metatarsophalangeal joint. Late features include: an osteochondral fragment, sclerosis and flattening of the bone and increased cortical thickening.

92
Q

4 A 42-year-old secretary complains of pain and tingling in the radial three and one-half digits of the palmar surface of her right hand. Her symptoms began almost a year ago with nocturnal burning and tingling and have progressed since then. Her thenar eminence shows early wasting. An MRI of her wrist is arranged for further assessment. What features would be expected?

a Increased signal intensity of the median nerve on T2WI

b Decreased signal of the median nerve on T1WI

C Dorsal bowing of flexor retinaculum

d Normal nerve diameter inside the carpal tunnel

e Thickening of the ligament of Struthers

A

4 Answer A: Increased signal intensity on T2WI

The pathogenesis of carpal tunnel is probably one of ischaemia with venous congestion and nerve oedema from anoxia of the capillaries followed by impairment of venous and arterial blood supply. There are multiple causes for carpal tunnel syndrome including physiological states such as pregnancy. Other radiological features include: `pseudoneuroma’ of median nerve, swelling of nerve within carpal tunnel, volar bowing of flexor retinaculum, tenosynovitis of tendon sheaths, enhancement of the median nerve due to oedema or alternatively no enhancement due to ischaemia from holding the wrist in a fixed flexed/extended position.

93
Q

3 A fit and well 72-year-old female presented to her general practitioner with medial knee pain, having twisted her knee four months previously. Initial films on attendance at the Emergency Department at the time of the injury were reported as normal. Repeat films several months later on showed flattening of part of the medial condyle and a radiolucent focus. Which of the following is the most likely diagnosis?

a Panner’s disease

b Metastasis

c Benign cortical defect

d Lymphoma

e Spontaneous osteonecrosis of the knee

A

3 Answer E: Spontaneous osteonecrosis of the knee (SONK)

This condition is most common in the seventh decade (range of 13-83 years) and occurs with acute pain. It is thought to be associated with a meniscal tear and microfractures (secondary to trauma), but there are multiple predisposing causes. An early bone scan (<5 weeks) would show pathology.
Findings on delayed films include:
* flattening of the weight-bearing area of the medial femoral condyle;
* a radiolucent focus in the subchondral bone;
* later, subchondral fracture and periosteal reaction, which can lead to osteoarthritis in around half.
Panner’s disease is osteonecrosis of the capitellum. The history is too acute for metastases or lymphoma. A benign cortical defect is more likely in first and second decades and is usually asymptomatic.

94
Q

26 A 17-year-old teenager was involved in a fight and presented to the Emergency Department with a severely bruised and swollen hand. What fracture is he most likely to have sustained?

a Fracture of the shaft of the fifth metacarpal

b Intra-articular fracture of the base of the first metacarpal

c Fracture of the scaphoid waist

d Rolando fracture

e Triquetral fracture

A

26 Answer A: Fracture of the shaft of the fifth metacarpal

This is often termed a `boxer’s fracture’. A Bennett’s fracture is an intra-articular fracture-dislocation of the base of the thumb metacarpal and Rolando fractures are intra-articular comminuted fractures of the base of the thumb metacarpal.

95
Q

33 A 35-year-old horse rider complains of severe pain in her left thigh after riding a horse for the first time in many weeks. She has recently returned from a long business trip where she was unable to do any horse riding or other physical activity while away. The clinical team request an ultrasound scan to confirm the clinical diagnosis. What is the most likely abnormality?

a Focus of low echogenicity within quadriceps tendon

b Discontinuity of sartorius tendon

C High echogenicity foci within gluteus maximus muscle belly

d Low attenuation within biceps femoris muscle belly

e Discontinuity of musculotendinous junction of adductor long-us muscle

A

33 Answer E: Discontinuity at musculotendinous junction of adductor long-us muscle

Acute adductor strain is caused by forceful adduction of the thigh during an abduction movement. This sudden change in direction requires powerful contraction of the adductor muscles, which may tear to varying degrees depending on the adequacy of prior stretching and the force of the movement. Unless there is a complete tear, loss of function will not result, though there may be pain, swelling and ecchymoses on the anteromedial aspect of the thigh. Ultrasound may confirm the presence of a tear, which is often seen at the musculotendinous junction.

96
Q

34 A 17-year-old girl attempted suicide by jumping from the second floor of her apartment building. She landed such that her left leg sustained almost all the impact of her fall. When seen in the Emergency Department she was fully alert, having sustained no head, chest, spinal or abdominal injuries. Her left leg was severely bruised with an open wound and the right leg and foot appeared normal. Radiographs revealed a comminuted distal tibial fracture with intra-articular involvement associated with a fracture of the mid-shaft of the left fibula. What type of injury is this?

a Maisonneuve fracture

b Torus fracture

C Osteochondral fracture

d Pilon fracture

e Lisfranc fracture

A

34 Answer D: Pilon fracture

A high-energy impact where the talus is driven into the tibia can result in a Pilon fracture. Alternatively, a fall from a height may lead to another pattern of injuries such as a comminuted fracture of the calcaneum or fractures of the pelvis or lumbar spine. A multiplanar CT examination is useful for operative planning to show the extent of the fracture.

97
Q

44 A 14-year-old keen hurdler presents with left hip pain and tenderness. A plain radiograph of the pelvis shows irregularity of the left anterior inferior iliac spine and an avulsed crescent of bone lateral to this. Which of the following is the most likely underlying cause of this?

a Avulsion of the hamstrings

b Avulsion of the iliopsoas muscle

C Avulsion of the rectus femoris muscle

d Avulsion of the sartorius muscle

e Avulsion of the gluteus medius muscle

A

44 Answer C: Avulsion of the rectus femoris muscle

Apophyseal avulsion injuries are more common in children than adults. Young athletes, in particular hurdlers and sprinters, are at particular risk.

98
Q

2 An elderly woman has been complaining of pain in her left hip for many months. The GP is eventually persuaded to request a hip X-ray to investigate this further as the pain is far more severe than her regular arthritis' pains. The report from the radiologist reads flattening and collapse of left femoral head consistent with avascular necrosis’. Which of the following is the most sensitive in diagnosing avascular necrosis (AVN) of the hip?

a Bone marrow imaging with radiocolloid

b Bone scan with diphosphonates

C Plain films

d MRI

e CT

A

2 Answer D: MRI

MRI is 90-100% sensitive and 80% specific for diagnosing symptomatic disease, reflecting death of marrow fat cells. Nuclear medicine scans are 80-85% sensitive, with bone marrow imaging more sensitive than bone imaging. Early disease shows a cold area from interrupted vasculature; late disease shows a `doughnut’ sign, a cold spot with a surrounding ring of increased uptake due to capillary revascularisation, and new bone synthesis. Plain radiographic appearances may be unremarkable in the early stages of AVN. Early diagnosis with MRI is therefore required if the diagnosis is suspected to prevent joint replacement.

99
Q

7 Following acute on chronic back pain a 61-year-old female with a history of steroid use for inflammatory bowel disease is referred for further investigation. Her pelvic and lumbar spine radiographs are unremarkable. An MRI shows patchy low signal in the first sacral segment with loss of height and a radionuclide bone scan shows sacral uptake in an `h-shaped configuration’ with mild degenerative changes elsewhere. What is likely diagnosis based on the above findings?

a Metastatic deposit

b Haemangioma

C Schmorl’s node

d Osteoporotic collapse

e Insufficiency fracture

A

7 Answer E: Insufficiency fracture

The `H’ sign is usually diagnostic of a sacral insufficiency fracture. The sacrum can be difficult to assess on conventional radiography and its absence does not exclude pathology.

100
Q

24 A young girl attended the Emergency Department after tripping over and falling onto her outstretched hand. Clinical examination revealed bruising and swelling on the palmar surface of her hand and she was exquisitely tender in the `anatomical snuff box’. What would be the best investigation to exclude a scaphoid fracture?

a Immediate plain radiography including scaphoid views

b Plain radiography including scaphoid views after 10 days

c Bone scan within 24 hours

d MRI within 24 hours

e Unenhanced CT

A

24 Answer D: MRI within 24 hours

There is controversy over whether MRI or scintigraphy are the most sensitive modalities for scaphoid fracture detection, but most authorities would accept that imaging within 24 hours is optimal and scintigraphy is most sensitive at 4-5 days post-injury.

101
Q

27 An elderly woman tripped on the edge of the curb and fell to the ground. She prevented herself from hitting her head by putting her right hand out in front of her. Her past history was unremarkable other than osteoporosis diagnosed on a DEXA scan two years previously. Immediately after the injury she noticed pain and restricted movement in her right arm. What injury is most likely to be visible on radiographs of the humerus and shoulder?

a Supracondylar fracture of humerus

b Posterior dislocation of shoulder joint

C Fracture of anatomical neck of humerus

d Fracture of deltoid tuberosity of humerus

e Fracture of surgical neck of humerus

A

27 Answer E: Fracture of surgical neck of humerus

The most likely injury in this context of falling on an outstretched arm, particularly in a patient with an osteoporotic humerus, is a fracture of the surgical neck, which is the point of weakness and is most easily fractured by an axial force. A fracture of the deltoid tuberosity is rare and would usually only occur with an avulsion by the deltoid tendon. The anatomical neck is not commonly fractured. Posterior dislocations are not common, occurring classically in the context of an epileptic fit or electrocution.

102
Q

30 A 19-year-old hockey player self-referred for physiotherapy for treatment of a long-standing knee problem. A recent radiograph had been reported: `There is flattening and cortical irregularity of the lateral surface of the medial femoral condyles bilaterally. Subchondral cysts and subchondral sclerosis are present, more marked on the right side.’ What is the most likely diagnosis?

a Osgood-Schlatter disease

b Osteoarthritis

C Juvenile chronic arthritis

d Psoriatic arthritis

e Osteochondritis dissecans

A

30 Answer E: Osteochondritis dissecans

Osteochondritis dissecans typically affects the lateral surface of the medial femoral condyle in adolescent males but may also involve the weight-bearing surface of the lateral femoral condyle, tibia or patella. A fragment may separate from the underlying bone and present as an intra-articular loose body (approximately 50% of cases) with pain, swelling and locking of the knee. The condition is bilateral in 20-25 % of cases. Osgood-Schlatter disease is the eponym for traction apophysitis of the tibial tuberosity. Subchondral sclerosis and cysts are hallmarks of degenerative arthritis, although the patient’s age makes this much less likely.

103
Q

34 A 45-year-old man with knee pain and instability after a ski injury is assessed in the Orthopaedic Clinic. Clinical examination reveals anterior knee instability with a positive anterior drawer and McMurray test. There is slight crepitus and joint line tenderness. AP and lateral radiographs of the knee are performed, which reveal a small bony fragment just lateral to the lateral tibial plateau. What is the most likely combination of findings?

a Medial meniscus and anterior cruciate ligament tear

b Lateral meniscus and anterior cruciate ligament tear and reverse Segond fracture

C Posterior cruciate ligament and joint capsule tears

d Sesamoid bone in biceps femoris tendon and ligamentous injuries

e Medial meniscus, anterior cruciate ligament tear and Segond fracture

A

34 Answer E: Medial meniscus, anterior cruciate ligament tear and Segond fracture

A Segond fracture is a capsular avulsion fracture seen at the lateral aspect of the proximal tibia and is often associated with high-energy sporting and ski injuries. There is usually associated ACL and meniscal injury and a medial tibial plateau impaction fracture may be present. A reverse Segond fracture is an avulsion fracture of the medial tibial margin. It is strongly associated with medial meniscal and PCL tears and caused by severe hyperextension and valgus stress.

104
Q

35 A teenager was brought to the Emergency Department after feeling the sensation of his right shoulder `popping out’. He was unable to use his right arm and radiographs confirmed an anterior dislocation. Which nerve should be specifically examined to exclude an associated injury?

a Musculocutaneous nerve

b Lateral plantar nerve

C Dorsal scapular nerve

d Radial nerve

e Axillary nerve

A

35 Answer E: Axillary nerve

Axillary nerve function is assessed pre- and post-shoulder relocation by testing for sensation in the `regimental patch’ area of skin in the upper arm and deltoid power (shoulder abduction). The nerve is at risk of damage from a dislocated humeral head where it lies close to the inferior joint capsule.

105
Q

65 A patient, a keen cross-country runner, was investigated for shin pain and plain radiographs appeared normal. The attending doctor was concerned they may have an occult stress fracture. Which investigation would be most sensitive?

a Repeat plain films after one week

b CT scan

C MRI scan

d Triple phase Tc99m methylene-diphosphonate (MDP) bone scan

e Gallium-67 white cell scan

A

65 Answer D: Triple phase Tc-99m methylene-diphosphonate (MDP) bone scan

Plain radiography is often used initially but generally only shows changes late on in the process, including endosteal and periosteal reaction. Triple-phase bone scintigraphy detects osteoblastic activity, which occurs during remodelling and is very sensitive. False positive results can occur in infection, osteonecrosis and neoplasia. CT is less sensitive than bone scintigraphy and MRI but it is useful in areas where plain film is limited and is more sensitive than plain film or MRI for detecting cortical fractures.

106
Q

67 An active 22-year-old man presented with shoulder pain following an injury playing rugby. He was found to have an anterior dislocation of his glenohumeral joint. What associated injury to the glenoid labrum is likely?

a Hill-Sachs lesion

b Reverse Hill-Sachs defect

C Bankart lesion

d Reverse Bankart lesion

e SLAP (Superior labral anterior to posterior) lesion

A

67 Answer C: Bankart lesion

This is a tear of the anterior glenoid labrum and may be associated with a Hill Sachs defect in the posterolateral surface of the humeral head.

107
Q

70 A 15-year-old athlete presented with pain in the left side of their pelvis and was noted to have a flake of bone adjacent to the anterior inferior iliac spine on their radiograph, which corresponded to the area of tenderness. What muscle are they likely to have avulsed?

a Adductor magnus

b Gracilis

c Psoas

d Rectus femoris

e Sartorius

A

d Rectus femoris

108
Q
  1. A 45-year-old man attends the Accident & Emergency Department with a 1-week history of foot pain. He is a regular runner and recently completed the London marathon. There is a history of lymphoma 15 years ago treated with chemotherapy. He is also diabetic and has chronic renal failure. Radiographs demonstrated a subtle periosteal reaction at the second metatarsal shaft. Bone scan shows focal tracer uptake in the second metatarsal region.
    What is the most likely diagnosis?

(a) Lymphoma deposit in second metatarsal

(b) Stress fracture

(c) Osteomyelitis

(d) Neuropathic foot

(e) Osteomalacia

A
  1. (b) Stress fracture

This is the most likely diagnosis as this is a very common site for stress fractures in the feet of runners, especially affecting the second and third metatarsals. Bone scan is almost 100% sensitive showing abnormal uptake within 6–72 hours of injury. MRI is also a very sensitive modality showing intermediate signal intensity on T1 and high signal on STIR and T2-weighted images.

109
Q

(Ped) 15. A 14-year-old girl presents after a twisting injury with inability to weight bear on the leg. Radiographs of the ankle and leg show a type III Salter–Harris fracture on the anteroposterior view. CT of the ankle confirms the fracture. The coronal reformats show that there is partial fusion of the medial part of distal tibial epiphyseal plate. What is the most likely diagnosis?

(a) Le Fort fracture

(b) Tillaux fracture

(c) Maisonneuve fracture

(d) Bennett fracture

(e) Pilon fracture

A
  1. (b) Tillaux fracture

This is a Salter–Harris type III fracture of the anterolateral distal tibial epiphysis resulting from an abduction and external rotation injury. This is seen in adolescents since the distal tibial epiphysis fuses in a medial to lateral direction and is not seen in adults where the growth plate has fused.
Pilon fractures are comminuted fractures of the plafond. Maisonneuve fracture involves tearing of syndesmosis, posterior malleolus, capsular injury and fracture of the proximal fibula. Le Fort fracture involves the distal fibula and the anterior tibiofibular ligament

110
Q
  1. A 60-year-old woman with recent history of falls presents with bilateral hip and low back pain. MRI of the pelvis shows bilateral sacral ala fractures and parasymphysial pubic fractures with marginal sclerosis. There is increased tracer uptake on bone scan of these regions. The most likely diagnosis is?

(a) Multiple myeloma with pathological fractures

(b) Insufficiency fractures

(c) Traumatic fractures

(d) Metastatic disease with unknown primary

(e) Lymphoma

A
  1. (c) Insufficiency fractures

Presence of bilateral sacral ala fractures with pubic fractures in this clinical context is virtually diagnostic of insufficiency fractures. Presence of marginal sclerosis at the fractures suggests chronic changes.

111
Q
  1. A 24-year-old man was involved in a road traffic accident. CT of the left knee shows isolated 5 mm depression of the lateral tibial plateau. What is the Schatzker classification of this fracture?

(a) Type 1

(b) Type 2

(c) Type 3

(d) Type 4

(e) Type 5

A
  1. (c) Schatzker type 3

Type 1 is lateral condylar split, type 2 is lateral split with depression, type 3 is pure lateral depression, type 4 is medial condylar fracture and depression, type 5 is bicondylar fracture and type 6 is fracture extending to metadiaphysis

112
Q
  1. A 13-year-old footballer complains of pain in the right groin after a tackle. Radiograph of the pelvis shows an avulsion fracture of the lesser tuberosity. Which muscle is attached to the lesser tuberosity?

(a) Iliopsoas

(b) Rectus femoris

(c) Sartorius

(d) Biceps femoris

(e) Vastus medialis

A
  1. (a) Iliopsoas muscle

This is attached to the lesser trochanter.

113
Q

(Ped) 25. A 14-year-old child injured his forearm in a rugby tackle. The radiograph shows a distal radial shaft fracture with dislocated distal radio-ulnar joint. What is the diagnosis?

(a) Galeazzi fracture dislocation

(b) Monteggia fracture dislocation

(c) Le Fort fracture

(d) Chopart’s fracture

(e) Essex–Lopresti fracture

A
  1. (a) Galeazzi fracture dislocation

Characterised by fracture of the distal shaft radius and dislocation at the distal radioulnar joint.

114
Q
  1. A 10-year-old boy presents after falling down stairs and sustaining injury to his left forearm. Radiographs show a displaced fracture of the proximal shaft ulna and anterior dislocation of the radial head. What is the diagnosis?

(a) Galeazzi fracture dislocation

(b) Monteggia fracture dislocation

(c) Essex–Lopresti fracture complex

(d) Weber fracture

(e) Maisonneuve fracture

A
  1. (b) Monteggia fracture dislocation

This involves a fracture of the proximal ulna shaft with dislocated radial head.

115
Q

(Ped) 26. A 15-year-old girl presents to the Accident & Emergency Department with pain in the left hip region after a recent half marathon. Radiography shows a small bony fragment inferior to the anterior inferior iliac spine. Which muscle has caused this avulsion?

(a) Sartorius

(b) Gluteus medius

(c) Rectus femoris

(d) Latissimus dorsi

(e) Iliopsoas

A
  1. (c) Rectus femoris is attached to the anterior inferior iliac spine.
116
Q
  1. A 62-year-old man presents with sudden-onset pain after minor injury. Plain radiograph shows subchondral sclerosis in the medial femoral condyle and joint effusion. MRI shows a diffuse oedema in the subchondral bone of medial femoral condyle with a crescentic linear fracture in a subchondral location. The likely diagnosis is?

(a) Spontaneous osteonecrosis of knee

(b) Osteoarthritis

(c) Osteochondritis desiccans

(d) Calcium pyrophosphate deposition disease

(e) Gout

A
  1. (a) Spontaneous osteonecrosis of knee

Typical subchondral fracture in elderly patient after minor knee injury.

117
Q
  1. A 40-year-old man with a history of dislocated left hip in a road traffic accident 2 years ago presents with left hip pain. Radiography show flattening and sclerosis in the superolateral part of the femoral head. The most likely diagnosis is?

(a) Degenerative arthritis

(b) Calcium pyrophosphate deposition disease

(c) Avascular necrosis

(d) Paget’s disease

(e) Prostatic metastases

A
  1. (c) Avascular necrosis

Avascular necrosis of the femoral head is a complication after hip dislocation. It may be seen as a wedge-shaped or geographical area of subchondral ischemic focus in the weight-bearing area. On MRI, there is a hyperintense inner border parallel to a hypointense periphery.

118
Q
  1. A 20-year-old runner presents with a history of right leg pain for 4 weeks. Radiography of the right leg shows a transverse cortical lucency and cortical thickening in the anterior cortex of the mid shaft of the tibia. What is the most likely diagnosis?

(a) Stress fracture

(b) Nutrient artery foramen

(c) Osteomalacia

(d) Normal variant

(e) Hypertrophic pulmonary osteoarthropathy

A
  1. (a) Stress fracture

This is commonly seen in runners and other athletes, in marching soldiers and in other patients in whom repetitive prolonged muscular action and stress are applied to a bone that is not accustomed to such action.

119
Q
  1. An 18-year-old football player presents with right groin pain after a tackle. The radiograph shows avulsion of the lesser trochanter. Which muscle is attached to the lesser trochanter?

(a) Sartorius

(b) Rectus femoris

(c) Iliopsoas

(d) Hamstrings

(e) Adductor longus

A
  1. (a) Iliopsoas

Iliopsoas is attached to the lesser trochanter.

120
Q
  1. A 42-year-old man in remission for lymphoma complains of bilateral hip pain. Coronal T1 images on MRI show geographical areas of abnormality in bilateral femoral heads, which are well demarcated from the normal bone by a thin rim of low signal on T1-weighted images. What is the most likely diagnosis?

(a) Lymphoma recurrence

(b) Red bone marrow

(c) Avascular necrosis

(d) Osteomyelitis

(e) Stress fractures

A
  1. (c) Avascular necrosis

This appearance on MRI is typical of avascular necrosis.

121
Q
  1. Which of the following correct regarding MRI of osteonecrosis (avascular necrosis) of femoral head: (T/F)

(a) Focal areas of low signal on T1W MRI suggest an alternative diagnosis.

(b) The “double line sign (adjacent high and low intensity line on T2W MRI) is virtually diagnostic.

(c) Fractures typically appear as low signal intensity lines on T1W images.

(d) Intravenous contrast should be administered routinely.

(e) MRI is less sensitive than isotope bone scan in making the diagnosis.

A

Answer:

(a) Not correct
(b) Correct
(c) Correct
(d) Not correct
(e) Not correct

Explanation:
Focal areas of homogeneous or inhomogeneous low signal on T1WI are commonly found. Contrast enhancement differentiates hypervascularized viable tissue from hypovascularized necrotic tissue. However it is not routinely done as it can be differentiated on other sequences. Isotope bone scan has a sensitivity of 80-85% and MRI has a sensitivity of 90-100%.

122
Q
  1. Which of the following are correct regarding trauma to the upper limb: (T/F)

(a) Non-or delayed union of humeral fracture is common.

(b) Bilateral shoulder dislocations are usually anterior.

(c) A fixed, abducted arm on shoulder X-ray is consistent with luxatio erecta.

(d) Humeral shaft fractures are associated with axillary nerve injury.

(e) The brachial artery is injured in 1-5% of humeral shaft fractures.

A

Answer:

(a) Not correct
(b) Not correct
(c) Correct
(d) Not correct
(e) Correct

Explanation:
Non-union or delayed union of humeral fracture is rare and occurs in cases secondary to excessive distraction of fracture fragments. Bilateral shoulder dislocation is rare and is usually posterior, mainly due to seizures. Humeral shaft fractures are associated with radial, medial or ulnar nerve injury. Radial nerve injury is most common (5-17%).

123
Q
  1. Which of the following are correct regarding knee trauma: (T/F)

(a) Anterior cruciate ligament (ACL) tears usually occur at the tibial attachment.

(b) “Deep lateral femoral sulcus” is a specific sign of posterior cruciate ligament (PCL) disruption.

(c) Kissing contusions at the anterior aspect of the proximal tibial and distal femur are characteristic of hyperextension injury.

(d) O’ Donoghue triad comprises complete disruption of the medial collateral ligament, ACL and posterior horn of medial meniscus.

(e) Dashboard injuries are typically associated with ACL tears.

A

Answer:

(a) Not correct
(b) Not correct
(c) Correct
(d) Correct
(e) Not correct

Explanation:
ACL tears usually occur in middle part of ligament followed by femoral attachment. Deep lateral femoral sulcus sign is highly specific for ACL disruption and results from impaction of the lateral femoral condyle on the posterior tibial plateau. Dashboard injuries are typically associated with midsubstance PCL tears at the genu

124
Q
  1. Which of the following are correct regarding osteochondritis dissecans:
    (T/F)

(a) Lesions in the knee are bilateral in 20-30%

(b) Lesions in the knee most commonly involve the lateral aspect of the femoral condyle.

(c) The average age of onset is within the 2nd decade

(d) A high signal intensity line around the lesions on T2W MRI images is indicative of instability

(e) A grade 2 osteochondritis dissecans lesions is characterised displaced a displaced fragment.

A

Answers:

(a) Correct
(b) Correct
(c) Correct
(d) Correct
(e) Not correct

Explanation:
Grading of osteochondritis dissecans:
Grade 1- focal softening/fissuring
Grade 2- defect in cartilage
Grade 3- fragment partially detached
Grade 4- displaced fragment/ loose body in the joint.